SlideShare uma empresa Scribd logo
1 de 106
Baixar para ler offline
CLINICAL CASES AND PEARLS

BENJAMIN BARANKIN, MD, FRCPC
ANIL KURIAN, MD
RAPID FIRE
DERMATOLOGY
Dr. Benjamin Barankin is a Toronto
dermatologist, the Founder & Medical Director of
Toronto Dermatology Centre
(www.torontodermatologycentre.com). He is also
the author of other dermatology books, including
Derm Notes, Stedman’s Illustrated Dictionary of
Dermatology Eponyms, Dermographies Volume 1 &
2, and Diagnostic Criteria in Dermatology.

DR. ANIL KURIAN is a dermatology resident at
the University of Alberta in Edmonton. At this early
stage in his career, he has already co-authored
more than 25 dermatology-related articles and also
received a prestigious award from the Ontario
Medical Association (OMA) for his patient advocacy
work promoting a ban on indoor tanning services to
minors.
Praise for Rapid Fire Dermatology
“Rapid Fire Dermatology by Drs. Barankin and Kurian is an
excellent state-of-the art guide to many of the common skin
disorders that primary providers will encounter with regularity.
The format differs from most other guides in that this book is
actually fun to read. It also provides realistic medication and
treatment options that are applicable to the primary care setting.
It will have broad appeal to its targeted audience.”
— Dr. David J. Elpern, Dermatologist, Williamstown, MA USA

“Rapid Fire Dermatology is a wonderful learning tool and
reference book for family physicians. Focusing on the most
common dermatological conditions seen in a family physician
office setting, this book offers clear pictures and an intriguing case
presentation approach that allows you to test your knowledge with
multiple choice questions. The explanations are clearly explained
with a wonderful practical approach to treating these
dermatological conditions. The treatment options are gauged at
the family practice level, and more advanced dermatological
approaches for more complex cases are discussed as well. It was
an enjoyable learning experience to read this book.”
— Dr. Monica Pearl, Family Physician, Toronto, ON Canada

“Clinical cases are a key component in modern medical education,
assisting the clinician to work through cases using best practice
techniques. Dermatology is a highly visual subject requiring the
health care provider to understand often very subtle differences in
the presentation of patients. This new book will help define
accurately the diagnostic and management criteria to base clinical
decision-making on. I am very excited about the potential for this
clinical reference tool!”
— Dr. Robert Norman, Dermatologist, Associate Professor,
Department of Dermatology, University of Central Florida
College of Medicine, Tampa Bay, FL USA

“Barankin and Kurian’s new book, Rapid Fire Dermatology, is an
outstanding work for those interested in learning the basics of
dermatology. The book focuses on some of the most common
afflictions of men and women, providing practical advice on both
differential diagnosis and management. The clinical photographs
are wonderfully illustrative of common disease presentations, and
the text is concise and easily read. If you are a family or other
primary care provider, a resident or a medical student, you will
see patients with these conditions. Rapid Fire Dermatology will
prepare you to recognize and manage them.”
— Dr. Steven R. Feldman, Dermatologist, Professor of
Dermatology, Pathology & Public Health Sciences, Wake Forest
University School of Medicine, Winston-Salem, NC USA
ii
“The case-based Rapid Fire Dermatology is a great way to learn a
wide variety of the most common skin diseases. The pictures are
terrific and the text is concise and to the point. I highly
recommend this book.”
— Dr. Gary Goldenberg, Dermatologist, Assistant Professor,
Dermatology and Pathology, Medical Director of the
Dermatology Faculty Practice, Mount Sinai School of Medicine,
New York, NY USA

“Drs. Barankin and Kurian have put together the perfect pocket
summary of the most common dermatologic conditions that a
family physician, pediatrician, or nurse practitioner will see on a
regular basis. Not only does the small book literally fit easily
within a typical white coat pocket, it details 25 of the most
common skin conditions seen in general practice using case
presentations and Q&A format that allows for the rapid
assimilation of the crucial information needed to make a definitive
diagnosis, initiate appropriate therapy, or recognize when a
referral to a dermatologist is warranted. This book is the ideal
entry level text to help demystify the confusing world of
dermatology to the primary care physician and other healthcare
providers.”
— Dr. Thomas G. Salopek, Dermatologist, Director, Associate
Professor, Division of Dermatology & Cutaneous Sciences at
University of Alberta, Edmonton, AB Canada

Copyright © 2012 Benjamin Barankin and Anil Kurian
All rights reserved. Except for the quotation of short passages for
review purposes, no part of this publication may be reproduced in
any form without prior permission of the authors.
Print Design: Brenda Conroy

iBook Design: Paul La Rue [Fission Media]
Printed Copies of this book may be obtained from:

Dr. Benjamin Barankin at benbarankin@gmail.com
Library and Archives Canada Cataloguing in Publication
iii
Authors
Dr. Benjamin Barankin is a Toronto dermatologist
specializing in medical, surgical, laser and cosmetic care of the
skin. His undergraduate university training was in Life Sciences
and Psychology at Queen’s University, followed by medical school
at the University of Western Ontario, and dermatology residency
at the University of Alberta.
Dr. Benjamin Barankin is also a widely published author with
more than 500 articles related to dermatology and he has gained
international distinction as the author of 6 other books on
dermatology. He has been honoured with being voted onto the
Executive Board of the Canadian Dermatology Association (CDA)
where he held the position of Secretary and Chair of the
Membership & Awards Committee. He serves on the editorial
boards of numerous medical journals and newspapers, and Dr.
Barankin is also team dermatologist for the Toronto FC soccer
team. He resides in Toronto with his physician wife and
daughters.

Dr. Anil Kurian is a dermatology resident at the University of
Alberta. Prior to entering medicine, he had a career in nursing,
working as a registered nurse and subsequently, a nurse
practitioner. He completed his Bachelor of Science in Nursing
degree from the University of Ottawa and his Master of Nursing
degree from Dalhousie University. He completed his medical
school training at McMaster University.
Dr. Anil Kurian is already making an impact in the field of
dermatology. At this early stage in his medical career, he has
already co-authored more than 25 dermatology-related articles
and also received a prestigious award from the Ontario Medical
Association (OMA) for his patient advocacy work promoting a ban
on indoor tanning services to minors. He is also project lead of
many groundbreaking dermatology projects in Edmonton,
including the innovative use of teledermatology in the ER
department, the first of its kind in Canada. He resides in
Edmonton with his family.

iv
Introduction
The idea of this book was developed through the very successful
case-based columns and articles written by the book’s senior
author, Dr. Benjamin Barankin, over the past decade. Dr.
Barankin has spoken at many primary care conferences and
aspires to educate family physicians about how important
recognizing and treating dermatological conditions are in a
primary care setting. Incorporating routine skin checks during a
patient’s physical examination promotes greater surveillance for
the diagnosis of pathological skin conditions and lesions, as well
as proper treatment being implemented in a timely fashion.
We aspired to integrate a large number of dermatological casebased conditions packaged into one succinct pocket-style book
with quality dermatology images. Each case will be described in
great detail and will provide the clinical practitioner with specific
tips on the types of dermatological visual and tactile cues they
should recognize and look for in each particular case, to improve
their diagnostic acumen. We envisioned a practical book
formatted to include exclusive Canadian medical content. We
believe that this book would be relevant to family physicians and
pediatricians, for use as a pocket reference in their clinic-based
practices, as well as medical residents and students wishing to
become familiar with various dermatological presentations.
Case-based scenarios are a well-received and utilized style of
learning among physicians and trainees. Dermatology is a highly
visual field within medicine and recognition of visual cues and
patterns, in conjunction with the clinical patient history, direct the
practitioner to the correct diagnosis. The 25 dermatology cases

presented within are common and important clinical concerns of
patients that include skin cancers, skin infections, inflammatory
and immunologic skin conditions, as well as benign lesions.
Additionally, each dermatology case will have an interactive
question and answer (Q&A) section where the reader can test
their knowledge relating to the clinical case presented. This
unique and interactive self-test will aid the reader to retain the
primary teaching points of each of the cases featured.
Rapid Fire Dermatology is a unique dermatology book tailored to
the Canadian marketplace, including up-to-date, comprehensive
information regarding dermatological diagnosis, as well as
specific medication and treatment recommendations.
Additionally, after each case presentation, there is an added
“Clinical Pearls” section with informative dermatologist tips
related to diagnosis or treatment aspects for that particular
condition. Rapid Fire Dermatology is a great resource for
Canadian physicians, medical residents or students. We hope you
find it of value to your practice.
If you have any comments about this book or suggestions for
topics for the next edition, please contact Dr. Benjamin Barankin
at: benbarankin@gmail.com

Acknowledgements
The authors would like to thank their respective families for their
unconditional support of educational pursuits by the authors, specifically
relating to dermatology education and research endeavours. We are also
grateful for the efforts of our book designer, Brenda Conroy.
The development and funding of this book has been made possible through
a grant provided by Leo Pharma Inc.
v
Question 1 of 3
1. What is her diagnosis?

A. Allergic contact dermatitis
B. Psoriasis
C. Tinea corporis
T HIGH RASH WITH CRUSTY LESIONS
A 15 year-old girl presents with a mild fever and a
moderately pruritic rash on her left and right thigh
regions. The lesions began 3 days before as blisters
discharging a clear yellowish fluid. The blisters
progressively enlarged despite treatment with a topical
antibiotic ointment. She is taking no other medications
and her medical history is unremarkable.

D. Bullous impetigo
E. Bullous pemphigoid

Check Answer

Diagnosis & Information
6
Predisposing factors include skin abrasions, minor trauma,
burns, poor hygiene, insect bites, diabetes mellitus, primary
varicella infection, and pre-existing skin disease, such as
eczema. Also, those with nasal or perineal Staphylococcal
aureus colonization are at increased risk for developing
impetigo.

Diagnosis: Bullous impetigo

Bullous impetigo commonly starts with small vesicles on the
face, trunk, buttocks, perineum, or extremities, without
surrounding erythema. These vesicles rapidly enlarge into
flaccid, transparent bullae measuring from 1 to 5 cm in
diameter. Bullae initially contain a clear yellow fluid that
subsequently becomes dark yellow and turbid. Their margins
are sharply demarcated, having no erythematous halo. The
lesions heal more quickly than nonbullous lesions. The
condition might be associated with fever, diarrhea, and
weakness, but unlike nonbullous impetigo it is not associated
with lymphadenopathy.

Impetigo is a superficial skin infection encountered most
frequently among children. It typically presents with multiple
vesicular lesions on an erythematous base, which eventually
crust over. There are 2 types: nonbullous and bullous. Both
forms involve only the most superficial layers of the skin.

The Nikolsky sign (sloughing of the epidermis with sliding
pressure) is usually negative. Complications of impetigo
include lymphadenitis, cellulitis, sepsis, or
glomerulonephritis.

The nonbullous form is the most common and is likely to be
caused by a mixed staphylococcal and streptococcal infection.
The bullous form, however, is a highly contagious condition
caused by an epidermolytic toxin (produced most commonly
by staphylococci) at the site of infection.

Cultures of fluid from an intact blister or moist plaque should
yield the causative agent; if the patient appears ill, blood
cultures should also be obtained. The differential diagnosis
primarily includes all of the herpes simplex viral infections,
allergic contact dermatitis, thermal burns, bullous insect bite
reactions, bullous pemphigoid and rarely pemphigus.
7
Prophylactic treatments to prevent staph infections include
‘bleach baths’. This is done by adding ¼ to ½ cup of common
liquid bleach to approximately 40 gallons of bath water.
Soaking in the bleach bath for about 10 minutes (2-3 times
per week over 1-2 weeks), is an effective way to prevent the
occurrence of cutaneous staphylococcal infections. This is
especially helpful in children with recurrent impetigo or
regular eczema flares.
Local treatment of cutaneous lesions can be achieved with 2%
mupirocin ointment (Bactroban®) or fusidic acid 2% cream
(Fucidin® cream) and sodium fusidate 2% ointment
(Fucidin® ointment). The former should be prescribed tid for
8-12 days, while the latter can be used 2-4 times daily for 8-12
days. In addition, local care, which includes cleansing,
removal of crusts, and application of wet dressings, is
sufficient to cure mild cases. Some studies have demonstrated
that 2% mupirocin ointment is as safe and effective as oral
erythromycin in the treatment of patients with impetigo,
while another study showed that 2% fusidic acid was more
effective than oral clindamycin, erythromycin or flucloxacillin
in the treatment of soft tissue infections. It is important that
duration of either individual topical therapy alone be limited
as bacterial resistance can be seen with both local treatment
options. Also, adding a mild topical steroid to antibiotic
therapy, such as fusidic acid plus hydrocortisone found in
Fucidin®H, is beneficial to deal with the underlying
dermatitis and pruritus.

Complicated, quickly spreading or widespread cases of
bullous impetigo require 5 to 10 days of oral β-lactamase–
resistant antibiotics, such as cephalexin (Keflex®)
250-500mg qid for 10 days or amoxicillin/clavulanate
(Augmentin®) 250-500mg bid for 10 days.

Clinical Pearl
• Impetigo is highly contagious; patients should not touch the
lesions or be discouraged by covering the lesion. The child
should be kept at home until the lesions resolve if in an
exposed area that other children may come in contact with.

“When you are looking for the cause of disease, think food, bugs
and drugs.” 

~ Drs. Walter & Dorinda Shelley, American Dermatologists and
authors of Advanced Dermatologic Diagnosis

8
Question 1 of 3
What is the most likely diagnosis?

A. Tinea corporis
B. Sarcoidosis
C. Granuloma annulare
R EDDISH - BROWN LESIONS ON DORSAL
HAND

A 36 year-old male presents with erythematous-brown
asymptomatic plaques on his dorsal hands. He is
bothered by the cosmetic appearance, and would like to
stop having to cover up the areas because of
embarrassment. He occasionally takes ibuprofen and
acetaminophen for tension headaches.

D. Lichen planus
E. Erythema annulare centrifugum

Check Answer

Diagnosis & Information
9
GA is an idiopathic condition, with controversial associations with
infections, diabetes, and thyroid disease; there appears to be a
hereditary component in some cases. Localized trauma and
sunlight have also been hypothesized to play a role in its etiology.
GA is often a clinical diagnosis. A punch or incisional biopsy is
recommended for atypical or uncommon presentations, and
especially if growing quickly and tender.

Diagnosis: Granuloma annulare
Granuloma annulare (GA) is a benign, inflammatory skin condition
of dermal papules and annular plaques commonly affecting acral
sites. GA occurs most commonly in women and in younger
individuals (typically less than 30 years old). There are several
clinical variants, including: localized GA (most common, especially
in children & young adults), generalized GA (10%), subcutaneous
GA, perforating GA (5%), and arcuate dermal erythema. Localized
GA typically manifests as groups of small erythematous to brown
papules, often in an annular arrangement (raised border, central
clearing) over distal extremities, especially the hands, feet, and
extensor limbs.

Management involves reassurance that the condition is benign and
spontaneous resolution of lesions typically occurs within 2 years in
50% of patients, especially of the localized clinical variant. Various
treatment options have been utilized, but often have suboptimal
clinical efficacy. Intralesional triamcinolone acetonide (Kenalog®,
2.5-5mg/ml q 4-6 weeks) is the most uniformly successful therapy.
Potent topical steroids with or without occlusion can also be tried
but have less efficacy. Cryotherapy in lighter skin types can also
provide benefit alone or more commonly in combination with
intralesional Kenalog®. While localized GA resolves after an
average of 2 years duration, generalized forms of GA typically have
a more chronic duration and unfortunately, are more resistant to
treatment options. Phototherapy and oral retinoids are commonly
employed in cases of generalized GA, and increasingly biologics are
being used.

Clinical Pearls
• Granuloma annulare can be a frustrating condition to
manage, as it is one of those conditions that develops for
reasons that are not well understood, and similarly often
resolves spontaneously.
• Interestingly, some cases have been reported to disappear
when oral antibiotics were given for sinus, fungal or other
infections.
10
“Just because you have a diagnostic handle on a disease doesn’t
mean you can handle it.” 

~ Drs. Walter & Dorinda Shelley, American Dermatologists and
authors of Advanced Dermatologic Diagnosis

11
Question 1 of 3
What is the most likely diagnosis?

A. Pilomatricoma
B. Epidermoid cyst
C. Milia cyst
S LOW GROWING BACK LUMP
A 64 year-old male presents to clinic with a slowly
growing nodule on his back. It has been present for at
least a couple years, is asymptomatic, and has never had
any bleeding or discharge. He has never had such a
lesion before.

D. Dermoid cyst
E. Lipoma

Check Answer

Diagnosis & Information
12
pilosebaceous unit, HPV infection, or traumatic implantation of
epidermal elements. A number of hereditary syndromes including
nevoid basal cell carcinoma syndrome and Gardner syndrome
have the feature of multiple epidermoid cysts. As well, idiopathic
scrotal calcinosis may represent the end stage (calcification) of
scrotal epidermoid cysts.
Patients often present to clinic for a number of reasons relating to
an epidermoid cyst. Reasons include the cosmetic appearance of a
visible body ‘lump’, the malodorous cheese/curd-like material that
can be extruded from the lesion, or because of lesional discomfort
associated with either inflammation or infection. Clinically, the
lesions appear as round, firm, flesh-coloured nodules (of various
sizes), which quite often have a central punctum. Commonly
affected areas on the body include the head, neck and trunk.

Diagnosis: Epidermoid cyst
Epidermoid cysts form as a result of the growth of epidermal cells
within a focal area of the dermis. The material inside the cysts is
chemotactic for neutrophils and can result in inflammation. Most
epidermoid cysts do not cause any problems and grow rather
slowly. However, cysts can become inflamed and less commonly
infected which results in redness, tenderness and pain, due to
trauma or other factors. Epidermoid cysts are more common in
men during the young adulthood years but may be noted at any
age.
The pathogenesis of an epidermoid cyst is not completely clear,
but some factors that may play a role include occlusion of the

The diagnosis of an epidermoid cyst is typically made clinically. If
infection of a cyst is of concern, a swab for bacterial culture can be
sent. Diagnostic imaging is rarely necessary unless the lesion is in
the midline of the body. In such a case, an ultrasound can be
considered.
Epidermoid cysts are benign lesions and require treatment only if
symptomatic or for cosmetic purposes. A tender, inflamed,
uninfected cyst often benefits from an intralesional triamcinolone
acetonide (Kenalog®) injection. For cysts presumed or proven to
be infected, incision and drainage is recommended following by a
course of oral gram positive coverage antibiotics (eg. Keflex®
500mg qid x 7-10 days).
Epidermoid cysts are excised using a classic large incision, mini
incision, or trephination with a punch biopsy device. Incising and
draining is a quick procedure, but has high recurrence risk since
13
the entire cyst wall is unlikely to be evacuated. Excision of the
entire cyst, including the epidermal cyst wall lining, is the most
definitive treatment, but even still recurrences may occur.

Clinical Pearl
• When removing epidermoid cysts, a punch biopsy instrument
can be used that can result in a smaller scar than doing an
elliptical excision. Along with trying to extrude the contents
with forceps and pressure and undermining, inserting and
moving back and forth a curette can be helpful in removing
any remaining cyst wall contents to prevent recurrence.

“Sometimes you have to wait for a disease to grow up before a
diagnosis can be made.”

~ Drs. Walter & Dorinda Shelley, American Dermatologists and
authors of Advanced Dermatologic Diagnosis

14
Question 1 of 3
What is the most likely diagnosis?

A. Pityriasis alba
B. Tinea versicolor
C. Post-inflammatory
hypopigmentation
W HITE PATCH OVER KNEE
A 40 year-old female presents with a two-year history of
a white patch on her anterior knee and lower leg area.
She has smaller white macules of this on her fingers and
ankle. These lesions are asymptomatic. She has a
history of pernicious anemia and a family history of
thyroid disease.

D. Vitiligo
E. Nevus anemicus

Check Answer

Diagnosis & Information
15
Screening for these co-morbidities with periodic blood tests
should be considered when a diagnosis of vitiligo is made.
Vitiligo progresses without symptoms, though early lesions may
occasionally be pruritic. Vitiligo presents as sharply demarcated
patches that are cosmetically disturbing to most patients,
especially those with darker skin colour where there is more
contrast. After a few small white lesions appear, they increase in
number and size, becoming confluent and having unusual shapes.
Vitiligo is classified as having localized or generalized
involvement. Although any area of skin can be affected, the most
commonly affected areas include: face, neck, bony prominences,
dorsal hands, and fingers. Hair in a patch of vitiligo will also
depigment. A Wood’s lamp examination can help clarify the
diagnosis as the depigmented white areas of vitiligo become
accentuated under the light.

Diagnosis: Vitiligo
Vitiligo is an acquired, benign, depigmentation of the skin caused
by destruction of melanocytes in skin and hair, resulting in white
macules and patches. It affects 1-2% of the population with an
equal gender distribution and most commonly onsets between
10-30 years of age. The etiology of vitiligo has not been fully
elucidated, but the pathogenesis is multifactorial, likely having
both genetic and autoimmune mechanisms. Melanocytes are
destroyed in the skin and those in hair follicles can also be affected
resulting in white hair. Vitiligo has been associated with other
autoimmune conditions such as: pernicious anemia, alopecia
areata, diabetes mellitus, Addison’s disease, and thyroid disease.

Management options include potent topical steroids and topical
calcineurin inhibitors (especially on areas with thin skin, such as
the face or intertriginous areas). Phototherapy is commonly
employed for more widespread involvement or resistant cases.
Less commonly, excimer laser, repigmentation surgery, and
depigmentation therapies are employed. A discussion of the use of
cover up makeup (e.g. CoverFX, Dermablend™, Covermark®)
and topical tanning creams should be considered. Management
can be frustrating as improvement is slow and unpredictable.

16
Clinical Pearls
• Vitiligo is a chronic and difficult condition to manage.
Fortunately, areas such as the face typically respond well to
treatment. Hands and feet are notoriously slow to respond to
therapy.
• Some dermatologists believe there is benefit to taking folic
acid, vitamin C, and vitamin B12 in the treatment of vitiligo.
Some have also observed improvement with long-term use of
ibuprofen or acetylsalicylic acid.

“The darker the skin, the more prominent the paler patches. 

But for people of every shade, vitiligo can cause distress as
strangers gawk and job interviews become sidetracked.”

~ Catherine Saint Louis, Journalist, New York Times

17
Question 1 of 3
What is the most likely diagnosis?

A. Compound nevis
B. Angiofibroma
C. Basal cell carcinoma
C HRONIC ERYTHEMATOUS FACIAL PAPULE
A 74 year-old female presents with an erythematous and
telangiectactic papule to her nose. She has been using
topical steroid and antifungal creams for the past 2
months with minimal clinical improvement.

D. Dermatofibroma
E. Squamous cell carcinoma

Check Answer

Diagnosis & Information
18
previous actinic keratosis. Chronic sun exposure in a fair skin type
is the major risk factor for the development of BCC. The most
common sites are the head (especially nose) and neck, trunk, and
arms and legs.
Clinical presentation can vary based on subtype, which include:
nodular BCC, superficial BCC, pigmented BCC, cystic BCC, and
morpheaform/sclerosing BCC. BCC is largely a clinical diagnosis
in many cases and no additional work-up is required. Diagnostic
accuracy is enhanced by good lighting and magnification with a
dermatoscope may be helpful. A biopsy of the lesion is indicated
when clinical doubt as to the diagnosis exists or when the
histological subtype of BCC may influence treatment selection and
prognosis.

Diagnosis: Basal cell carcinoma (BCC)
This is the most common malignancy in humans, typically
appearing on sun-exposed skin. BCCs can result in significant
local destruction and even disfigurement, especially if located
around the nose or eyes. They rarely metastasize (<0.1%). In
Caucasians, there is approximately 25-35% lifetime risk of
developing a BCC. These lesions are uncommon in dark skin
types. Likelihood of BCCs increases with age, and they are
considered uncommon under 35 years of age.
Risk factors for BCC include a fair complexion, red or blond hair,
light eye color, increased sun exposure during childhood years,
ionizing radiation, UVA radiation, immunosuppression and

BCC is a highly treatable form of skin cancer. Treatment of BCC
can be surgical or nonsurgical. Surgical approaches with most
definitive 5-year cure rates include surgical excision and Mohs
micrographic surgery (99%) followed by curettage and
electrodessication and cryosurgery (95%). Nonsurgical
approaches include topical imiquimod, radiotherapy, and
photodynamic therapy. Preventive measures for BCC include
avoidance of the sun during peak hours and protection against sun
exposure with a hat, clothing, and broad spectrum sunscreens.
Avoiding tanning salons is also a must.

Clinical Pearl
• A patient with a history skin cancer complaining of a “sore
that doesn’t heal” or “the lesion has scabbed and/or bled on
a few occasions but won’t go away” is highly suggestive of
the diagnosis of BCC.
19
“Inject xylocaine or saline into a suspect basal cell carcinoma to
enhance its gross morphology.” 

~ Drs. Walter & Dorinda Shelley, American Dermatologists and
authors of Advanced Dermatologic Diagnosis

20
Question 1 of 3
. What is your diagnosis?

A. Melasma
B. Lentigines
C. Post-inflammatory hypermelanotic
macules
D ARK PATCHES ON CHEEKS
A 42 year-old woman presented with brown macules
that she has had for several years on both cheeks. She
began to notice the macules during her pregnancy of her
first child 4 years ago. She currently takes no
medications, and her medical history is otherwise
unremarkable.

D. Addison’s disease
E. Discoid lupus erythematosus

Check Answer

Diagnosis & Information
21
with darker complexions, including those of Hispanic, Middle
Eastern, Asian, and Indian ethnicities.
While most cases are idiopathic, melasma has also been
associated with genetics, exposure to ultraviolet radiation,
pregnancy, oral contraceptives, hormone replacement therapies,
mild endocrine disturbances (e.g. ovarian or thyroid dysfunction),
cosmetics, antiepileptic medications, and other photosensitizing
drugs. Sun exposure tends to be the most important causative
factor in all patients, so management options for melasma must
always include avoidance of excessive exposure to sunlight.

Diagnosis: Melasma
Melasma is a common hypermelanosis that typically occurs on
sun-exposed areas of the face. Melasma is characterized by
symmetrically distributed macules with irregular borders ranging
from light brown to dark brown to gray-brown. Common clinically
distinct distributions include centrofacial (forehead, nose, chin,
upper lip), malar (cheeks), and mandibular (lower jaw). Forearms
are also occasionally affected. Pigmentation typically slowly
evolves over many weeks or months.
Melasma is rare before puberty and is noted more commonly in
women of reproductive age. Men comprise approximately 10% of
the affected population. Melasma is more prevalent among people

Wood’s lamp examination is useful in diagnosis and can help
classify melasma into epidermal, dermal, and mixed
types. Epidermal pigmentation is the most common type of
melasma where pigmentation is accentuated under Wood’s light
examination. Dermal pigmentation decreases under Wood’s light
examination, and mixed types show areas with increased
(epidermal), absent, or decreased (dermal) pigmentation.
Classifying the melasma type is a valuable indicator of prognosis.
While epidermal pigmentation tends to respond better and faster
to hypopigmenting agents (eg. hydroquinone), dermal
pigmentation lesions show variable response, often with much
less patient satisfaction.
Melasma is a cosmetic problem that is difficult to treat. Current
treatments include hypopigmenting compounds, exfoliative
agents, and lasers. Topical hypopigmenting agents, such as
hydroquinone 2% to 10%, arbutinin, lumixyl, retinoic acid or
tretinoin, azelaic acid, vitamin C, mild-to-moderate strength
topical corticosteroids, and combinations of these agents are
effective. Exfoliative agents, such as alpha-hydroxy acid chemical
peels or microdermabrasion, are useful adjuncts to therapy. In
22
recent years, various laser therapies have been used with mixed
results.
In some cases, melasma has spontaneously resolved several
months postpartum or after cessation of oral contraceptives. If
indicated, discontinuation of causative medications is appropriate.
Treatment and prevention of melasma should always include
avoiding excessive sun exposure and daily use of broad-spectrum
sunscreens.

Clinical Pearls
• Granuloma annulare can be a frustrating condition to
manage, as it is one of those conditions that develops for
reasons that are not well understood, and similarly often
resolves spontaneously.
• Interestingly, some cases have been reported to disappear
when oral antibiotics were given for sinus, fungal or other
infections.

“If you haven’t looked at it with the makeup off, 

you haven’t looked at it.” 

~ Drs. Walter & Dorinda Shelley, American Dermatologists
and authors of Advanced Dermatologic

23
Question 1 of 3
What is your diagnosis?

A. Psoriasis
B. Nummular eczema
C. Allergic contact dermatitis
I TCHY BUMPS TO LOWER ABDOMEN
A 36 year-old male presents with a pruritic papular
eruption to his lower abdomen. He had mild acne as a
teenager, but no other skin problems. He has not used
any topical agents to treat his abdomen and takes a
proton-pump inhibitor (PPI) to treat his
gastroesophageal reflux.

D. Tinea corporis
E. Fixed drug eruption

Check Answer

Diagnosis & Information
24
contact dermatitis”) as opposed to a specific sensitivity, in the case
of ACD.
Common causes of ACD include nickel, fragrances, preservatives,
black hair dye, topical medications (e.g. neomycin, bacitracin),
latex, rhus, cobalt, chromate, epoxy resin, rubber accelerators,
rosin and many more chemical agents. Patients often complain of
significant pruritus, and may have mild burning or discomfort.
Lesions are typically edematous, erythematous, and welldemarcated.

Diagnosis: Allergic contact dermatitis
Allergic contact dermatitis (ACD) is a delayed type of induced
sensitivity that occurs in individuals that have been previously
sensitized to any of more than 3,000 chemicals, and is more likely
in genetically predisposed individuals. The allergic reaction causes
inflammation of the skin that is manifested by varying degrees of
erythema, edema, and vesiculation. A reaction typically develops
within 24-48hrs of a subsequent exposure, although it can occur
up to 7 days later; it typically resolves after 2-3 weeks. The term
“contact dermatitis” sometimes is used incorrectly as a synonym
for ACD. Contact dermatitis is inflammation of the skin induced
by chemicals that directly damage the skin (also known as “irritant

Definitive treatment is the identification and removal of any
potential causal agents; otherwise, the patient is at increased risk
for chronic or recurrent dermatitis. Referral for skin patch testing
(not skin prick testing; typically performed by a dermatologist) is
appropriate. Online resources allow the physician to create a list
of products free of allergens to which the patient is allergic, and
should be avoided. Potent topical steroids are used, and
occasionally if the reaction is severe or more widespread, a short
course of oral steroids (e.g. Prednisone) can hasten resolution.
Cool saline compresses, sedating antihistamines to aid in sleep,
and topical immunomodulators (e.g. Protopic®, Elidel®) and
emollients can provide benefit to the patient as well.

25
Clinical Pearls
• A pruritic rash localized to the infra-umbilical area is an
allergic contact dermatitis to nickel until proven otherwise.
If this rash is not treated, it can eventually spread out to
other areas of the body in an immune phenomenon termed
the “id reaction.”
• If a jean snap is a trigger for nickel contact (common), apply 1
coat of red nail polish on the inside of the snap and also
encourage tucking in of one’s shirt. Once the red nail polish
has disappeared (e.g. typically after 2 washings/dryings of
the jeans), the absence of the red colour will remind you to
repaint the jean snap. You can also cover it with duct tape.

“In a particular clinic, the incidence of allergic contact
dermatitis is determined by the interest the dermatologist takes
in allergic contact dermatitis.” 

~ Drs. N. Hjorth & S. Fregert, Swedish and Danish
Dermatologists

26
Question 1 of 3
What is the most likely diagnosis?

A. Epidermoid cyst
B. Milia cyst
C. Digital mucous cyst
U NCOMFORTABLE FINGER LESION
A 50 year-old male is bothered by the appearance of a
papule on his finger. It has grown very slowly over many
months and doesn’t bleed. The lesion is tender only
when it is firmly palpated or when he “accidentally
bumps it.”

D. Verruca vulgaris
E. Cystic basal cell cancer

Check Answer

Diagnosis & Information
27
yellow-tinged. The lesions, which are usually solitary and appear
lateral to the midline, are more common on dominant hands. Toes
are less commonly involved.
Although usually asymptomatic, pain can occur if there is
impingement on nerve fibers, and rarely larger cysts can disfigure
the affected digit. Nail deformities occur in many cases – called
the “nail groove” sign. Women are more commonly affected (more
than twice as often as men), as are those individuals between the
ages of 50 and 70.

Diagnosis: Digital mucous cyst
Digital mucous cysts (DMCs) are benign and often asymptomatic
ganglion cysts of the digits, typically located at the distal
interphalangeal (DIP) joints or at the proximal nail fold. The
etiology is uncertain, although it may relate to mucoid
degeneration of connective tissue; this process seems to involve
communication with the adjacent DIP joint and often is associated
with osteoarthritis, and less commonly with antecedent trauma.
Digital mucous cysts are translucent to flesh-coloured, solitary,
round and dome-shaped semi-firm papulonodules. The cysts can
be from a few millimeters to approximately one centimeter in
diameter. The cysts contain a gelatinous thick fluid that is clear or

Surgery or pseudo-surgical options are the standard of therapy
and attempt to remove the cyst and any underlying stalk
connected to the joint. Using conservative therapies, one can
periodically needle the cyst with a wide-bore needles resulting in
drainage and scarification until the cyst resolves. Aspirating the
cyst contents followed by intralesional triamcinolone acetonide
(10mg/ml) or liquid nitrogen cryotherapy is commonly employed.
Less commonly, curettage +/- electrodessication, CO2 laser, or
sclerotherapy techniques can be utilized.
More aggressive surgical procedures using cold-steel surgical
excision have been used which may require flaps or grafts.
Another approach involves marsupialization, or excision of the
whole proximal nail fold, with subsequent healing by secondary
intention.
Aggressive surgery to remove underlying osteophytes at the joint
results in few recurrences. However, this procedure may slightly
increase the risk of surgical complications (e.g. joint stiffness,
reduced movement). Osteophyte removal alone (without removal
of the cyst) has been reported to be effective. Nail deformities
often resolve following osteophyte removal. While osteophyte
28
removal or open joint procedures are usually performed in the
hospital setting, other management techniques can be performed
in an office setting by primary care providers and dermatologists.

Clinical Pearl
• Puncturing a few holes into the lesion with a 25guage needle,
extruding most of the viscous contents, and then applying
cryotherapy is effective in many cases. As a backup, the
patient can be given the needle that was used and instructed
that if there is a recurrence, to boil the needle and repuncture the cyst and squeeze out its contents, eventually
leading to permanent scarification and an end to the cyst.

“Do not show your wounded finger, for everything will knock
up against it.” ~ Baltasar Garcian, Spanish philosopher

29
Question 1 of 3
What is your diagnosis?

A. Pigmented basal cell carcinoma
B. Melanoma
C. Dysplastic nevus
D. Congenital melanocytic nevus
D ARK SPOT ON CHEEK
A 41 year-old woman presents with a dark lesion to her
right cheek. The lesion causes no discomfort and it does
not bleed or itch. The patient is not exactly sure how
long the lesion has been present, but feels it has been at
least a few months. She has a personal history of mild
facial acne as a teenager and has no family history of
skin problems. The patient worked as a lifeguard when
she was a teenager and recalls several painful sunburns.
She does not take any medications and has no known
drug allergies.

E. Blue nevus

Check Answer

Diagnosis & Information
30
respectively. However, mortality rates are higher in darker skin
types due often to a more advanced presentation. There is little
gender discrepancy. Older persons are more likely to acquire and
die from melanoma, with a median age at diagnosis of 53.
Primary risk factors for or clinical warning signs of melanoma
include the following:
•	

	


Changing mole (most important clinical warning sign; 

the “E in Evolution”)

•	


Clinical atypical/dysplastic nevi (particularly >5-10)

•	


Large numbers of common nevi (>100)

•	


Large (giant) congenital nevi (>20 cm diameter in an adult)

•	


Previous melanoma

•	


Sun sensitivity/history of excessive sun exposure

Diagnosis: Melanoma

•	


Melanoma in first-degree relative(s)

Melanoma is a skin cancer of pigment-producing melanocytes that
typically affects the skin, and less commonly the mucosae, eyes,
leptomeninges and gastrointestinal tract. It account for
approximately 5% of skin cancers, but 75% of deaths from skin
cancer. It is believed that two-thirds of melanomas arise de novo,
while one-third develop from pre-existing nevi. In Canada,
approximately 5,000 people are diagnosed with melanoma each
year, and approximately 950 will die of the disease. Detection and
treatment in early stages provides the best opportunity for cure.

•	

	


Prior nonmelanoma skin cancer (basal cell and squamous 

cell carcinoma)

•	


Male sex

•	


Age older than 50 years

•	

	

	

	


Presence of xeroderma pigmentosum or familial atypical 

mole melanoma syndrome: These 2 genodermatoses confer a

500- to 1000-fold greater relative risk of developing

melanoma.

Caucasians are most commonly afflicted, with black and Hispanic
persons having one twentieth and one-sixth the prevalence,

31
The most common warning signs are a new or changing mole.
Variation in the ABCDs (Asymmetry, Border irregularity, Color
variegation, Diameter >6mm) is evident in more than 80% of
lesions at diagnosis. Less commonly, one may note symptoms
such as pruritus, pain, bleeding or ulceration. The letter “E” has
been added to the ABCDs to signify “evolution” or change in an
existing lesion. As well, the “ugly duckling” warning sign which
signifies concern for a lesion that looks different from the rest is a
beneficial adjunct to the ABCDE rule.
The four main clinico-histopathologic subtypes of primary
cutaneous melanoma are: superficial spreading melanoma (most
common on trunk in men, and legs of women), nodular melanoma
(15-30% of cases), lentigo maligna melanoma (slow-growing,
usually head & neck of elderly), acral lentiginous melanoma
(occurs on palms, soles or under nail plate; more common in
darker skin types).

Clinical Pearls
• Any new black lesion on the skin should be strongly
considered for biopsy or referral to a dermatologist.
• Even after melanoma wide excision, these patients and those
persons in their genetic lineage should be screened
periodically for life and a much higher index of suspicion for
biopsy is warranted for any new lesion or dark lesion on their
skin.

“It is easy to know a nevus. But to know a melanoma takes
suspicion and excision, and remember that close observation
can help prevent a lentiginous lesion from becoming a
‘litigenous’ one.” 

~ Drs. Walter & Dorinda Shelley, American Dermatologists and
authors of Advanced Dermatologic Diagnosis

If detected early, melanoma can be cured with surgical excision.
Stage 1 lesions (melanoma in situ) confer a 5 year survival rate of
95%, whereas, stage IV melanoma metastatic to skin,
subcutaneous tissue or lymph nodes with normal LDH has a 5year survival rate of 19%. A stage IV melanoma that has
metastasized to lungs has a 5 year survival rate of 7%. Most
metastases occur within the first three years of a melanoma
diagnosis, the most common locations for metastases are: lymph
nodes, lung, liver, bone and brain.
Patients diagnosed with melanoma must be followed for life not
only because of risk of recurrence of their melanoma, but also
because they are at increased risk of another primary melanoma.

32
Question 1 of 3
What is the most likely diagnosis?

A. Large congenital melanocytic nevus
B. Becker’s nevus
C. Melanoma
D. Acanthosis nigricans
L ARGE BROWN HAIRY PATCH
A 19 year-old male presents with a several year history
of an increasingly evident hyperpigmented and
hypertrichotic area to his left upper chest. The lesion is
asymptomatic. He is healthy and takes no medications.

E. Tinea versicolour

Check Answer

Diagnosis & Information
33
in the majority of cases a Becker’s nevus is considered a benign
process. However, there have been a small number reported cases
of melanoma within a Becker’s nevus. The diagnosis is usually
clinical, although a skin biopsy can provide a histopathological
diagnosis and helps distinguish it from other clinical entities, all of
which are benign.
Patients should be informed that this is a benign entity that does
not require treatment except for cosmetic reasons. Patient
concerns relate to the hyperpigmentation and/or the
hypertrichosis. At present, it is quite difficult to remove the
hyperpigmentation, although hypertrichosis can be safely and
effectively removed by using laser hair removal or electrolysis.

Clinical Pearl

Diagnosis: Becker’s nevus
A Becker’s nevus is an organoid nevus with hamartomatous
elements, not a melanocytic nevus, predominantly affecting
males. The nevus typically first appears as an irregular
pigmentation (hyperpigmentation) on the torso or upper arm
(though other areas of the body can be affected), and gradually
enlarges irregularly, becoming thickened and often with
hypertrichosis.
There is an association with ipsilateral hypoplasia of breast and
skeletal anomalies including scoliosis, spina bifida occulta, or
ipsilateral hypoplasia of a limb that has been termed the Becker
nevus syndrome, a sporadic condition. Other than the syndrome,

• This benign late appearing birthmark is most common on the
upper trunk and patients should be reassured as to its
benign nature, and that currently we can clear the
hypertrichosis, but that we are likely a few years away from
being able to remove the hyperpigmentation with laser.

“The skin is a diagnostician’s delight. Everything to be named is
in full view. One has but to look, and recognize.” 

~ Drs. Walter & Dorinda Shelley, American Dermatologists,
Advanced Dermatologic Diagnosis

34
Question 1 of 3
What is your diagnosis?

A. Alopecia secondary to irondeficiency
B. Androgenetic alopecia
C. Alopecia areata
R OUND , BALD PATCHES ON SCALP
A 29 year-old male is concerned with three round areas
of hair loss to his posterior scalp that he has had for the
previous two months. The areas are asymptomatic and
he is otherwise healthy, though he hasn’t had a blood
test in years. He had a similar round lesion on his scalp
years ago that resolved on its own over a few months.

D. Telogen effluvium
E. Tinea capitis

Check Answer

Diagnosis & Information
35
round, well-defined, bald patches with “exclamation-mark hairs”
at the periphery. AA has an unpredictable onset but most
commonly affects the scalp. However, any of the hair-bearing
areas of the body can be affected, including the eyebrows,
eyelashes, beard, or even the body. Interestingly, there may also
be nail pitting (similar to that found in psoriasis). It is typically
asymptomatic, though occasionally mildly pruritic preceding the
lesion development. Entire scalp involvement is termed alopecia
totalis, while hair loss affecting the entire body is termed alopecia
universalis.

Diagnosis: Alopecia areata
Alopecia areata (AA) is a non-scarring autoimmune condition that
most commonly presents in young adulthood. AA, which affects
men and women equally, affects 1.5-2% of the general population
at some point in their lives. Most people with AA are healthy
individuals, though they do have a higher risk of atopy, and
autoimmune conditions such as thyroid disease and vitiligo;
family members may have autoimmune conditions such as
insulin-dependent diabetes, rheumatoid arthritis, thyroid disease
or systemic lupus erythematosus.
Although not life threatening, it can have a profound psychosocial
impact on individuals. Lesions clinically present as one or more

The differential diagnosis of AA should include other non-scarring
alopecias such as telogen effluvium, androgenetic alopecia, and
even tinea capitis. Occasionally a scalp biopsy or referral to a
dermatologist is required to ascertain the diagnosis. Other
autoimmune conditions including atopic dermatitis, vitiligo,
thyroid disease, and pernicious anemia have been associated with
AA, as has iron deficiency, and thus are often screened for on
blood tests.
The treatment of choice is intralesional triamcinolone acetonide
(Kenalog®, 2.5-5mg/ml) injected in multiple sites 1cm apart on
the affected areas every 4-6 weeks until regrowth. Hair growth
usually becomes apparent in four weeks. Local skin atrophy, the
predominant side effect, can be minimized by taking care to inject
into the mid-dermis, rather than into the more superficial
epidermis or the lower subcutaneous fat. Potent topical steroids
are also used, as well as topical minoxidil (Rogaine®) and contact
sensitizers such as diphencyclopropenone (DPCP) for more
widespread involvement. Hair prostheses should be
recommended for widespread involvement.

36
Clinical Pearl
• In children with alopecia areata, in light of the often selflimited nature of this condition and the pain factor, starting
treatment with a potent topical steroid often makes more
sense than going straight to intralesional cortisone.

“Hair brings one’s self-image into focus; it is vanity’s proving
ground. Hair is terribly personal, a tangle of mysterious
prejudices.”

~ Shana Alexander, former Journalist and Commentator on 60
Minutes

37
Question 1 of 3
What is your diagnosis?

A. Impetigo
B. Angular cheilitis
C. Hand, foot and mouth disease
D. Herpangina
S ORES AT CORNER OF MOUTH
A 24 year-old female presents with painful lesions at the
corner of her mouth. Every few months, she develops
tingling, burning and erosions to the area and is quite
frustrated with topical antiviral therapy.

E. Herpes simplex

Check Answer

Diagnosis & Information
38
such as those lining the eyes, vagina, cervix, or inside of the
mouth. The skin around the blisters is often red.
The diagnosis of HSV infection may be made clinically, but
laboratory confirmation is recommended in patients presenting
with primary or suspected recurrent infection. The gold standard
of diagnosis is viral isolation by tissue culture, although this
process can take as long as four to five days, and the sensitivity
rate is only 70% to 80%. Despite these limitations, viral culture is
still the diagnostic test of choice for HSV skin infections.
Serologic testing is extremely sensitive but is not helpful during
primary infection because of the delay in antibody development.
Polymerase chain reaction enzyme-linked immunosorbent assay
(PCR-ELISA) is extremely sensitive (96%) and specific (99%) but
expensive. For this reason, it is not used for the diagnosis of skin
lesions but is the laboratory test of choice for diagnosing HSV
encephalitis.

Diagnosis: Herpes simplex
Herpes simplex virus (HSV) is a DNA virus presenting as grouped
vesicles on an erythematous base. Typically infections will be
recurrent in the same location. While oral herpes labialis is
usually caused by HSV-1, genital involvement is more commonly
due to HSV-2, although we now see both strains in both locations.
Transmission occurs when an actively shedding individual (with
clinical lesions or not) contacts mucous membranes which may be
open or abraded. The virus then travels to sensory dorsal root
ganglion where latency is established. Various stimuli can elicit
outbreaks, including: trauma, fever, stress, hormonal fluctuations,
and immunosuppression (e.g. from UV radiation). An eruption of
tiny vesicles appears on the skin or on the mucous membranes,

Susceptible patients without existing antibodies to HSV develop
primary infection after first exposure to HSV. Any recurrence of
HSV is termed “recur-rent infection.” About 20% to 40% of the
general population experience recur-rent infection of the lips or
perioral region.
Herpes viruses establish lifelong infections and the virus cannot
currently be eradicated from the body. Treatment usually involves
antiviral drugs (acyclovir, valacyclovir, famciclovir; valacyclovir
(Valtrex®) has the highest percentage bioavailability at 70%) that
interfere with viral replication, reducing the physical severity of
outbreak-associated lesions and lowering the chance of
transmission to others. Resistance to antiviral drugs in
immunocompetent individuals is rare. Treatment is most effective
39
if started early (within 12 hours of the start of symptoms such as
burning or tingling sensation is ideal, although within 72 hours is
part of the formal indication) and can help symptoms resolve a
day or two sooner (“episodic therapy”). Studies of vulnerable
patient populations have indicated that daily use of antivirals can
reduce reactivation rates. For people who have frequent, painful
attacks, reactivation rates can be reduced by continuous therapy
(“chronic suppressive” or “maintenance therapy”) with antiviral
drugs.
Docosanol (Abreva®) is an over-the-counter cream used for
recurrent oral-facial herpes simplex episodes (cold sores or fever
blisters). It is used 5 times per day for up to 10 days. Acyclovir 5%
cream (Zovirax®) is a prescription antiviral topical agent used to
treat cold sores. It is used 5 times per day for 4 days once tingling
sensation or erythema is noticed to the mouth region. Oral
antiviral therapy is considered significantly more effective than
topical therapy. Analgesic medications are also commonly used to
manage painful lesions.

Clinical Pearl
• Episodic therapy is sufficient for patients with only the
occasional herpes simplex eruption, though for patients with
frequent eruptions or who want to lessen the risk of
transmission to others, chronic suppressive therapy is
advised.

“An inefficient virus kills its host. A clever virus stays with it.” ~
Professor James Lovelock, Independent scientist and ecologist
40
Question 1 of 3
What is the most likely diagnosis?

A. Verruca vulgaris
B. Squamous cell carcinoma
C. Basal cell carcinoma
D. Prurigo nodularis
R APIDLY EVOLVING HAND PLAQUE
A 78 year-old female presents with a three month
history of a rapidly growing large plaque with central
crust on her dorsal hand, and which is occasionally
tender. She has mild hypertension, but is otherwise
healthy. She has no history of skin cancer, but has had
“things burned off.”

E. Keratoacanthoma

Check Answer

Diagnosis & Information
41
KAs typically affect elderly Caucasians, especially men. Several
variants of this tumor exist, including: eruptive keratoacanthoma
of Grzybowski (multiple non-involuting KAs), and multiple
Ferguson-Smith keratoacanthoma (rare, autosomal dominant,
self-healing, affecting young adults).
Treatment options are largely surgical. If left untreated, many KA
lesions can resolve spontaneously over a period of a few months.
However, due to a small metastatic potential as evidenced by rare
instances of metastases or invasion (possibly misdiagnosed SCC),
surgical excision of the lesion is warranted in many cases.
Occasionally systemic retinoids are employed for multiple KAs.
For poor surgical candidates or those with lesions that due to size
and location are difficult to excise, intralesional methotrexate, 5FU or bleomycin can be effective. Aggressive liquid nitrogen
cryotherapy can also be considered and may need to be repeated.

Clinical Pearls

Diagnosis: Keratoacanthoma (KA)
A keratoacanthoma (KA) is a common, low-grade tumor that can
clinically resemble a squamous cell carcinoma. Some experts
believe KA to be a variant of squamous cell carcinoma, and thus
suggest similar treatment. A keratoacanthoma is characterized by
its rapid growth velocity (over weeks to months). There is often a
solitary, firm skin to red-coloured papule with a central
crateriform keratin plug (“volcano” appearance). Etiologic factors
for this tumor are multifold, including: ultraviolet radiation,
immunosuppression, smoking, human papilloma virus, tar or
pitch exposure, trauma, and genetic factors.

• Although some KAs can self-resolve even if left alone,
because the lesion is often unsightly or uncomfortable, and
because of the potential that the lesion is actually a
squamous cell carcinoma, KAs are typically treated for cure.
Additionally, the scar left after spontaneous resolution is
typically not as cosmetically acceptable as a surgical scar.
• A rapid growing scaly tumour that looks like a mini-volcano is
likely a KA.


“People don’t get checked enough. What scares me is that a lot of
people with skin cancer came in for something different and I
just happened to see the tumor. ” 

~ Dr. Quang Le, American Dermatologist
42
Question 1 of 3
What is the most likely diagnosis?

A. Lichen planus
B. Lichen simplex chronicus
C. Pityriasis rosea
D. Pustular psoriasis
I TCHY PAPULES ON WRISTS
A 33 year-old male presents with numerous pruritic
papules on his wrists, shins and ankles. The patient
finds the rash very bothersome and recently has
difficulty sleeping. He takes occasional migraine
medications, and has no drug allergies.

E. Nummular eczema

Check Answer

Diagnosis & Information
43
Lesions slowly develop over weeks and months, often starting on
the wrists. Pruritus is common and worse in the hypertrophic
form (often on the shins). Oral lesions can be asymptomatic or
cause burning and pain. Most cases will resolve within two years,
although oral involvement often implies a longer duration. Along
with cutaneous involvement, scalp (lichen planopilaris can cause
scarring and permanent alopecia), nails (10% of patients; nail
plate thinning, grooving and ridging, scarring), oral (tongue,
buccal) and genitals (vulva, glans) can be affected.

Diagnosis: Lichen planus
Lichen planus (LP) is a pruritic papulosquamous disease of
planar, polygonal, and purple/violaceous papules (5 P’s).
Common sites of involvement include the flexor surfaces of the
arms and wrists, shins, ankles, genitalia, and oral mucosa (lacy
white “Wickham striae”). There is no gender or racial predilection,
and while LP can occur at any age, those age 30-60 years-old are
most commonly afflicted. Several clinical subtypes have been
observed: hypertrophic, atrophic, erosive, follicular, annular,
linear, vesiculobullous, lichen planus pemphigoides, and actinic.
The diagnosis of LP is often clinical, although histopathology may
be required to confirm the diagnosis.

LP appears to be an idiopathic immunologically-mediated
inflammatory disorder. Stress may play a role in triggering this
condition. Different variants exist, which have different
presentations and symptomatology. Hypertrophic LP is
particularly pruritic, and often affects the shins. The oral erosive
form of lichen sclerosus has been associated in some cases with
Hepatitis C viral infection. Ulcerative lesions of the mouth and
vulva, unlike the cutaneous lesions, have a small risk of malignant
transformation into squamous cell carcinoma. Other conditions
with altered immunity may co-exist with LP, including alopecia
areata, vitiligo, lichen sclerosis and myasthenia gravis.
LP is a self-limited condition that usually resolves within 1-2
years. Most patients are treated symptomatically and with potent
topical steroids. Oro-genital involvement can benefit from topical
calcineurin inhibitors, such as tacrolimus (Protopic®). Less
commonly, oral metronidazole (Flagyl®), oral acitretin
(Soriatane®), phototherapy, and systemic steroids are employed
by dermatologists for resistant or generalized LP.

44
Clinical Pearl
• Both psoriasis and lichen planus commonly present in men
on the glans penis (remember the P’s of lichen planus:
purple, polygonal, planar, pruritic papules….and penis!).
While STDs are important to rule out in lesions affecting the
glans, remember these two immunologic conditions as well.

“Ordinarily, lichen planus is as easily recognizable as an old
friend. He greets you with an itch and wears iridescent purple
patches in all the old familiar places on wrist and ankle….then
with experience you will learn to recognize your friend by his
masks. You will come to know him sans itch, sans papule, and
sans Wickham’s striae.” 

~ Drs. Walter & Dorinda Shelley, American Dermatologists and
authors of Advanced Dermatologic Diagnosis

45
Question 1 of 3
The most likely diagnosis is?

A. Tinea corporis
B. Cutaneous lupus erythematosus
C. Chronic form of nummular
dermatitis
S CALY , ERYTHEMATOUS RASH ON ELBOWS
A 63 year-old woman presented with a scaly,
erythematous rash on her elbows and buttocks. She has
been using daily moisturizers and 1% hydrocortisone
cream without much clinical improvement. Her past
medical history includes hypertension and
dyslipidemia. She is currently taking ramipril
(Altace®), atorvastatin (Lipitor®), and a multivitamin
on a daily basis.

D. Psoriasis vulgaris 

(mild-to-moderate type)
E. Mycosis fungoides

Check Answer

Diagnosis & Information
46
About 35% of patients with psoriasis have a family history of the
disease. Several environmental factors can trigger psoriasis in
susceptible individuals: infection (most commonly streptococcal
infection); trauma to the skin (surgery, sunburn, scratching;
known as “Koebner phenomenon”); drug reaction (e.g., lithium,
beta blockers, anti-malarial drugs, non-steroidal antiinflammatory drugs, and glucocorticoids); and stress.

Diagnosis: Psoriasis vulgaris 

(mild-to-moderate type)
Psoriasis is a chronic, recurring inflammatory disease that can
affect the skin, scalp, nails and joints. The typical lesions are
pruritic, erythematous, and present as well-demarcated papules
and plaques with silvery-white scales. Psoriasis affects about 2%
of the population and ranges in severity from mild to severe;
patients with moderate to severe disease can experience a
significant impact on quality of life. It affects men and women
equally. The age of onset of psoriasis follows a bimodal
distribution (peaks between ages 20 to 30 years and again
between the ages of 50 to 60). Both genetic and environmental
factors have been implicated in the pathophysiology of psoriasis.

The clinical presentation of psoriasis varies depending on the
morphologic subclass. Plaque psoriasis (Psoriasis vulgaris) is the
most common subtype and is usually concentrated on the
extensor surfaces (i.e., elbows, knees, and lumbar back), scalp,
genital areas, palms, and soles. Removal of scale causes sites of
punctate bleeding (Auspitz sign), a diagnostic sign of historic note.
Therapy varies depending on disease severity, as determined by
the degree of body surface area involvement (BSA). Mild psoriasis
involves <2% of the BSA, moderate psoriasis covers 2% to 10% of
the BSA, and severe psoriasis covers >10% of the BSA. However,
the vast majority of patients (approximately 80-90%) presents
with relatively mild disease and have only limited involvement of
the skin, which can be well-controlled with topical therapy alone.
Patients with scalp, face, genital, and palmoplantar involvement
in particular can have a very significant impact on their quality of
life.
Diagnosis of psoriasis is mainly made on clinical grounds. History
and physical examination are usually diagnostic and should
include a screen for arthritis because up to 30% of patients with
psoriasis have or will develop joint involvement (psoriatic
arthritis). Skin biopsy can be helpful in difficult cases. The
differential diagnosis includes pityriasis rosea, seborrheic
dermatitis, lichen simplex chronicus, tinea infections,

47
psoriasiform drug eruptions, mycosis fungoides, and nummular
eczema.
Mild-to-moderate psoriasis is typically treated using topical
agents. Useful topical preparations include those including
corticosteroids, vitamin D analogues, combinations of these, and
calcineurin inhibitors. Corticosteroids remain the mainstay of
topical treatment for mild-to-moderate psoriasis. In choosing an
appropriate corticosteroid potency and vehicle, the physical
location being treated and patient preference should be
considered. Psoriatic patients with thick, chronic plaques often
require treatment with the highest potency corticosteroids (+/LCD tar or salicylic acid) and/or intralesional triamcinolone
acetonide.
Topical vitamin D modulators are among the most widely used
medications for the treatment of mild-to-moderate psoriasis.
Dovonex® (calcipotriol ointment and cream) and Silkis®
(calcitriol 3 µg/g) ointment are synthetic topical vitamin D
analogs with a great safety profile. Dovobet® (calcipotriol/
betamethasone proprionate) ointment combines the benefits seen
in both a topical vitamin D analog and a topical corticosteroid.
Mechanistically, this refers to the anti-proliferative/ prodifferentiating effects of the vitamin D analog component and the
anti-inflammatory effects of the corticosteroid component.
Clobex® spray (clobetasol propionate) is another good and
elegant option for plaque psoriasis.

of calcineurin inhibitors in some patients are a burning sensation
and pruritus with initial treatments; however, the discomfort
generally diminishes with ongoing use.
The aforementioned topical therapies can be applied once or twice
daily for up to 8 weeks. If insufficient improvement is noted,
consider referral to a dermatologist.
For recalcitrant mild-to-moderate or for more extensive psoriasis,
psoralen plus ultraviolet-A topical therapy administered 2 to 3
times a week or more commonly narrow-band (311 nm)
ultraviolet-B therapy are effective options and typically covered
under provincial plans.
Several systemic agents have been used as therapy for moderate
psoriasis not controlled by topical therapy alone. Methotrexate,
acitetrin, and less commonly cyclosporine are most widely used.
Rotating ultraviolet light and systemic medications can help
prevent toxicity from continuous use of any one therapy alone.
Increasingly, biologic therapies (e.g. adalimumab, etanercept,
infliximab, and ustekinumab) are also being used for moderate
plaque psoriasis.

There are two topical preparations of calcineurin inhibitors:
Protopic® (tacrolimus ointment 0.03% and 0.1%) and Elidel®
(pimecrolimus cream 1.0%). Tacrolimus and pimecrolimus have
been used in areas of skin where greater topical penetration is
improved, such as in flexural or facial skin. The main side-effects
48
Clinical Pearls
• Make sure to use the right vehicle in the right location. For
instance, for the scalp or other hair bearing areas, lotions,
foams or gels are great vehicles, while for very thick or
fissured plaques of psoriasis, an ointment is a better fit.
Also, ask your patients if they have a specific preference.
• Psoriasis skin is very thick, so patients need to understand
that although “steroids thin the skin,” in psoriasis this is a
very wanted side effect!
• For patients asking about more natural or lifestyle options,
there is evidence to support the benefits of weight reduction,
smoking cessation, reducing alcohol, and consuming more
fish or fish oil capsules.
• For patients with a significant impact on their quality of life,
consider more aggressive therapeutic options and/or referral
to a dermatologist.

“Psoriasis is an acronym for a Pitting, Scaling, Ongoing, Red,
Inflammatory, Anguishing, Symmetrical, Inherited Skin
disease.” 

~ Drs. Walter & Dorinda Shelley, American Dermatologists and
authors of Advanced Dermatologic Diagnosis

49
Question 1 of 3
What is the most likely diagnosis?

A. Fordyce glands
B. Flat warts
C. Xanthelasma
D. Syringomas
P ERIOCULAR PAPULES
A 34 year-old female is bothered by the appearance of
papules under her eyes that she has had for several
years. She mentions that an aunt had similar
asymptomatic, flesh-coloured papules which increased
in number over time. She is healthy and takes no
medications.

E. Milia

Check Answer

Diagnosis & Information
50
include the axilla, chest, abdomen, and genitals. Syringomas have
been associated with Down syndrome.
Where clinical uncertainty as to a diagnosis exists, a skin biopsy or
referral to a dermatologist can be beneficial.
Patients should be reassured as to the benign nature of these
lesions. Treatment is primarily for cosmetic reasons only, with the
goal of having complete destruction of lesions without evidence of
scars. Syringomas can be treated with local excision,
electrodessication, dermabrasion or cutaneous resurfacing with a
carbon dioxide or erbium-yag laser.

Clinical Pearl

Diagnosis: Syringomas

• Syringomas are benign, cosmetically unappealing skincoloured to yellowish small papules most commonly found in
females, especially in my experience in Asian skin. The lower
eyelids have many more lesions than the upper eyelids.

Syringomas, considered benign adnexal tumors, are fairly
common lesions more commonly affecting females, and
predominantly of cosmetic concern. Lesions typically first appear
around the time of puberty.

• Even with treatment, patients should be warned that lesions
can recur and/or new lesions may develop over time, so
multiple treatments would be typical over one’s lifetime.

Syringomas are asymptomatic flesh-colored to yellow small
dermal papules that can also appear translucent. The lesional
surface is either flat-topped or round, and the lesions typically are
quite small, having a diameter of less than 4mm. Syringomas are
distributed symmetrically in clusters, usually involving the lower
eyelids and upper cheeks. Other less commonly affected areas

“You’ll never get the big picture without looking at the little
details.” ~ Drs. Walter & Dorinda Shelley, American
Dermatologists and authors of Advanced Dermatologic
Diagnosis

51
Question 1 of 3
What is the most likely diagnosis?

A. Verruca vulgaris
B. Papular dermatitis
C. Molluscum contagiosum
D. Corns
F INGER PAPULES

E. Acquired digital fibrokeratomas

A 32 year-old male presents with a seven month history
of papules growing and multiplying on his hand. They
are asymptomatic, but unsightly.
Check Answer

Diagnosis & Information
52
Warts are transmitted by direct or indirect contact, and disruption
of the epithelial barrier is a predisposing factor. A subset of HPV
types has been associated with the development of malignancies:
HPV 6, 11, 16, 18, 31, and 35; malignant transformation is most
common in those with genital warts or those who are
immunocompromised. There are now two HPV vaccines
authorized for use in Canada: Gardasil® and Cervarix®.
Gardasil® is approved for use in females and males aged 9-26,
and provides protection against four HPV types: HPV-6, HPV-11,
HPV-16 and HPV-18. Cervarix® was recently approved for use in
Canada for females aged 10 to 25. At this time Cervarix® has not
been approved for use in males in Canada. Cervarix® provides
protection against the two HPV types: HPV-16 and HPV-18.

Diagnosis: Verruca vulgaris (Warts)
Cutaneous warts, one of the most common skin diseases, are a
frequent presenting complaint to family practice and dermatology
clinics. Warts are benign proliferations of skin and mucosa
confined to the epithelium caused by one of the more than 150
types of the double-stranded DNA human papilloma virus (HPV).
Common, plantar and flat warts are cutaneous manifestations of
the human papillomavirus (HPV). Certain HPV types tend to
occur at particular anatomic sites; however, warts of any HPV type
may occur at any site. Clinically, HPV manifests itself as either:
common warts, flat warts, genital warts, or palmoplantar warts.

The treatment of warts poses a therapeutic challenge
for physicians as no general consensus on monotherapy to achieve
complete remission in all patients has been reached. As a result,
many different approaches to wart therapy exist with varying
mechanisms of action and efficacy.
A ‘watch-and-wait approach’ is the first option to consider, since
65% of warts are believed to regress spontaneously within two
years. Non-treatment in patients with extensive, spreading, or
symptomatic warts is not advised. The first line of therapy is the
topical agents, particularly salicylic acid, which is effective for
non-genital warts. Many salicylic acid preparations are available
over the counter (e.g. Soluver plus®, Duofilm™) at
concentrations up to 40%, can be applied at home, and have cure
rates from 70-80%. Dermatologists may prescribe even stronger
compounded acid mixtures containing trichloroacetic acid,
salicylic acid and lactic acid. Topical vitamin A acid/retinoids can
also be considered in the therapeutic armamentarium.

53
There are several topical agents that are applied only in the
physician’s office, and they include catharidin and powerful
sensitizing agents such as diphencyprone in a treatment referred
to as immunotherapy. Cryotherapy destroys warts by thermal
cytolysis. It can be used to treat warts in any location. An
experienced physician applies liquid nitrogen with a cotton
applicator or the more aggressive cryospray. Cryotherapy has been
shown to be superior in the treatment of common warts. 5fluorouracil (5-FU), such as Efudex®, is a chemotherapeutic
agent that interferes with DNA and RNA synthesis, and both
topical and intralesional compounds have been used to treat
genital warts. Imiquimod (Aldara®, Zyclara®) has also been used
for hand warts with some success, although it is more useful for
mucosal warts where absorption is much greater. For persistent
and refractory warts, intralesional injections, most commonly
Bleomycin or candida antigen, can be performed by an
experienced physician.

Clinical Pearl
• Make clear to your patients that there is no quick and easy fix
as far as warts are concerned.
• Evidence-based medicine for non-mucosal warts best
supports liquid nitrogen cryotherapy and salicylic acid
preparations.
• Consider oral zinc therapy as an immune booster for
stubborn warts – start at 50mg/day with food (studies show
it can be increased as tolerated up to 600mg/day).

“Of all the futile disorders of the skin, it would be hard to find
any that are regarded with greater contempt by the lay public
and yet capable of resisting a greater variety of treatment than
the group of papillary lesions commonly known as warts.” 

~ Dr. W.W. Lempiere, Australian Dermatologist

The treatment of warts continues to pose a therapeutic challenge.
Many different approaches to wart therapy exist with varying
mechanisms of action and efficacy. The treatment options are
extremely heterogeneous and no single treatment or group of
treatments is standard. Patients are often dissatisfied with
recurrence, as a therapeutic regimen to achieve complete
remission in all patients has not yet been developed. Additional
clinical trials of the many therapeutic options will help to clarify
the best treatment choices for cutaneous warts.

54
Question 1 of 3
What is the most likely diagnosis?

A. Subungual wart
B. Psoriatic nail
C. Onychomycosis
D. Traumatic nail changes
T HICK , YELLOW TOENAILS
A 56 year-old male has slowly changing nails over the
past 10 years. They have gotten thicker, yellow in
colour, and occasionally cause him discomfort. He finds
cutting his toenails to be increasingly difficult.

E. Pseudomonas nail infection

Check Answer

Diagnosis & Information
55
dermatophyte molds. However, the clinical appearance is
indistinguishable between the different fungal species. The main
subtypes include: distal lateral subungual onychomycosis (DLSO),
white superficial onychomycosis (WSO), proximal subungual
onychomycosis (PSO), and candidal onychomycosis.
Clinically, onychomycosis manifests with an increased nail
thickness, an opacified nail plate, subungual thickening, and/or
onycholysis. There is also often discoloration of the nail ranging
from white to yellow to brown in colour. Diagnosis is usually
clinical, although the nail changes of psoriasis or lichen planus
and the effects of aging can have a similar appearance. Further
testing such as direct microscopy with 20% potassium hydroxide
(KOH) testing can be beneficial, and a fungal culture is often used
to identify the causative organism.

Diagnosis: Onychomycosis
Onychomycosis is a fungal infection that most commonly affects
the toenails. It can affect any component of the nail unit, including
the nail matrix, nail bed, or nail plate. The majority of the time,
the concern is primarily cosmetic in nature, but occasionally,
there can also be discomfort and pain.
The prevalence of onychomycosis is approximately 6% and it
accounts for half of all nail disorders. Onychomycosis is
uncommon in childhood, but is seen with prevalence rates up to
90% in the elderly. The most common microorganism implicated
is the dermatophyte Trichophyton rubrum. Onychomycosis is
caused by 3 main types of fungi: dermatophytes, yeasts, and non-

Topical therapy is beneficial in mild cases involving the very distal
nail plate and when few nails are involved. Topical therapy is also
considered in combination with systemic therapy or where the
patient is on multiple oral medications. The best topical therapy
option is likely ciclopirox olamine 8% nail lacquer solution
(Penlac®). Ciclopirox lacquer is usually applied to the affected
nails once daily at bedtime or at least 8 hours before washing the
affected area. Apply daily over the previous coat. Once a week,
remove the lacquer with rubbing alcohol. Loose nail material
should be filed and nails trimmed, as required. Treatment of
onychomycosis (especially multiple nails or of long duration) is
largely systemic with terbinafine (Lamisil®) considered by most
to be the drug of choice. Less commonly, itraconazole pulse
therapy or nail avulsion is employed. Not treating the nails is a
common and important option to discuss.

56
Clinical Pearls
• Onychomycosis and tinea pedis are often important to treat
in diabetic patients since the fissures or maceration caused
by tinea can provide an entry point for bacteria, and
ultimately risk for cellulitis, sepsis, or amputation.
• For patients with recurrent tinea cruris (“jock itch”), always
check their toenails as there is often a concomitant
onychomycosis.

“You can tell a lot from a person’s nails. When a life starts to
unravel, they’re among the first to go.” 

~ Ian McEwan, critically-acclaimed author and Man Booker
Prize winner

57
Question 1 of 3
What is the most likely diagnosis?

A. Scalp psoriasis
B. Severe seborrheic dermatitis
C. Mycosis fungoides
S CALY , ERYTHEMATOUS PLAQUES ON HEAD
A 54 year-old man comes to your clinic with a
longstanding history of ‘dandruff’ that is pruritic and he
is unable to get under control. He has tried using many
shampoos advertised on television and recommended
by his pharmacist but has seen no clinical improvement.
He also has scaly plaques to his elbows and knees. He is
otherwise healthy and takes no medications.

D. Tinea capitis
E. None of the above

Check Answer

Diagnosis & Information
58
white or gray scale and may extend beyond hair margins, affecting
the forehead, neck, ear and retroauricular area. Quality of life can
be severely compromised, particularly if lesions are visible, scales
fall onto clothes, and when pruritus is intense. Hair loss secondary
to poorly managed scalp psoriasis can occur. Thick accumulated
scale and hair, particularly with very thick hair, may be obstacles
to medications reaching the scalp.
The diagnosis of scalp psoriasis is most often made clinically. In
some cases, it may be difficult to differentiate between scalp
psoriasis and seborrheic dermatitis, but the presence of psoriasis
in other locations may aid its diagnosis in the scalp. Localization
of disease past the scalp margins and sharply demarcated plaques
may help differentiate scalp psoriasis from seborrheic dermatitis,
as seborrhea usually does not extend beyond the scalp margins
but may be found in the central facial region or central chest.

Diagnosis: Scalp psoriasis
Psoriasis affects approximately 2% of the population, and in
50-80 percent of cases, the scalp is affected. Scalp involvement
may occur in isolation, with plaque psoriasis located elsewhere
(most common), or with erythrodermic, pustular or guttate
psoriasis, and/or psoriatic arthritis. In 25-50 percent of affected
patients, psoriasis presents initially on the scalp. Family history
may predispose patients to scalp psoriasis.
Mild forms of scalp psoriasis involve minimal scaling, while severe
disease may be associated with thick, crusted plaques that are
frequently associated with moderate to severe itch. Classic scalp
lesions are asymmetric, sharply demarcated, covered with silver-

Scalp psoriasis is a chronic inflammatory disease and cure is not a
realistic treatment expectation. Instead, the clinician’s focus
should be on enhancing the patients’ skin-related quality of life.
Non-pharmacological physical therapies which include moist
occlusion to plaques and gentle debridement of scalp scaling are
useful in some patients. Occlusion therapy involves first applying
skin products, such as moisturizers to the skin then wrapping the
skin with plastic wrap and tape. Occlusion helps keep the area
moist and increases the effectiveness of medicated creams.
Topical agents are the mainstay of pharmacologic treatments,
with clinically proven efficacy. Scalp psoriasis can be managed
with topical preparations of tar and salicylic acid, such as Sebcur/
T™ (10% tar with 4% salicylic acid) and Neutrogena® T/Gel®
(0.5% tar) medicated shampoos, Derma-smoothe/FS® scalp oil
(fluocinolone topical oil) overnight with shower cap, Dermovate®
59
lotion (clobetasol 17 - propionate), or Diprosalic® lotion (0.05%
betamethasone and 2% salicylic acid). The retinoic acid derivative
Tazorac® (0.1% tazarotene gel), applied nightly, can be helpful for
scalp psoriasis, but must be used in combination with steroids to
prevent irritation.
Newer topical agents for scalp use are being used currently with
better patient satisfaction and compliance. Dovobet® gel is a new
lipophilic gel specially formulated for the scalp and contains the
active ingredients calcipotriol 0.005% (synthetic vitamin D
analogue) and betamethasone dipropionate 0.05% (potent
corticosteroid).” Clobex® (Clobetasol proprionate) 0.05%
shampoo is also efficacious and safe for the management of
moderate scalp psoriasis; it is often prescribed twice weekly
(alternating with other psoriasis shampoos) and effectively helps
to prevent the relapse of scalp psoriasis; this 15 minute shortcontact shampoo formulation can be safely utilized for extended
periods and weaned as necessary.
Topical therapies may be used in combination with sunlight or
ultraviolet light (phototherapy) for moderate psoriasis that affects
less than 20% of the skin surface. Brief exposures to ultraviolet B
light (UVB), and less commonly UVA, improves psoriasis and
treatments are usually conducted three times per week.
As an alternative to topical application, corticosteroids
administered through intralesional injection (e.g. Kenalog®
(triamcinolone acetonide), 5-10 mg/mL) to recalcitrant scalp
lesions may provide some patients with a reduction of symptoms,
often lasting several months. Systemics used for management of
psoriasis include cyclosporine (least common), methotrexate and
acitretin; and although these therapies may be effective, each
involves inherent limitations. Biologics have become an important

modality for the treatment of moderate-to-severe chronic plaque
psoriasis and may be an effective approach to management of
difficult plaque psoriasis of the scalp. Current biologics on the
market for psoriasis include the tumour necrosis factor-alpha
blockers Enbrel® (etanercept), Remicade® (infliximab), and
Humira® (adalimumab). One of the newest biologics is Stelara®
(ustekinumab), a human monoclonal antibody directed against
interleukin (IL)-12 and IL-23. Many new biologics are in the
pipeline.

Clinical Pearls
• For stubborn scalp psoriasis, consider multi-modal
treatment, meaning shampoo and overnight gel or lotion use.
For very stubborn plaques, consider intralesional Kenalog®.
• Scalp psoriasis can have a tremendous impact on quality of
life and can be difficult to control, and so occasionally
systemics or biologics are employed even if this is the only
area affected by psoriasis. Phototherapy can also be
considered, especially in those persons with little or no hair.

“I have long had the idea that God created psoriasis and warts
to teach dermatologists the merits of humility.”

~ Vincent Joseph Derbes, Dermatologist and former Editor,
International Journal of Dermatology

60
Question 1 of 3
What is the most likely diagnosis?

A. Basal cell carcinoma
B. Actinic keratosis
C. Keratoacanthoma
D. Irritated seborrheic keratosis
L ARGE SCALY SCALP PLAQUE
A 67 year-old male presents with a scaly plaque on his
scalp. It has slowly enlarged in the past year, and is
otherwise asymptomatic. He has a history of actinic
keratoses, and worked in construction.

E. Squamous cell carcinoma

Check Answer

Diagnosis & Information
61
involvement and invasion of dermis). The diagnosis of squamous
cell carcinoma is based on clinical suspicion and confirmed by a
histopathological diagnosis on biopsy.
Treatment options include surgical excision, as well as curettage
and electrosurgery (similar cure rates). For well-differentiated
small SCCs (<2cm), a 4mm margin is recommended, whereas in
larger tumors in high risk sites require at least a 6mm margin.
Larger SCCs and those that are poorly differentiated have a higher
incidence of local invasion and metastases. Mohs micrographic
surgery is indicated for aggressive subtypes of SCC, recurrent
tumors, or highly aggressive features (e.g. SCC in a burn scar).
Other treatment options include radiation and cryosurgery in
patients with impaired clotting disorders or tumors in difficult to
excise areas.

Diagnosis: Squamous cell carcinoma
(SCC)
Squamous cell carcinoma is currently the second most common
skin cancer in Caucasians, and the most common type in black
individuals. SCC can develop in the setting of actinic keratoses,
leukoplakia, lichen sclerosus, radiation dermatitis, chronic arsenic
exposure, scars, burns, chronic ulcers or sinuses (e.g. hidradenitis
suppurativa).

Prevention of SCC is similar to preventative treatment of actinic
keratoses (AKs). Effective measures include avoidance of
excessive sun exposure, especially during peak day-time hours,
avoidance of artificial sources of ultraviolet (UV) light such as
tanning beds or prolonged UV treatments, wearing protective
clothing, applying sunscreen with at least 30 SPF over sunexposed areas a few times per day. Additionally, proper
management of AKs and aggressively managing other underlying
conditions (e.g. lichen sclerosus) is important.

There is thought to be a spectrum of keratinocyte malignant
transformation: actinic keratoses (atypical keratinocytes in up to
one-half of the epidermis), Bowen disease or SCC-in-situ (full
epidermal keratinocyte atypia), and invasive SCC (full epidermal
62
Clinical Pearls
• An SCC on a dark-skinned individual will often present on the
lower leg as a benign-appearing hyperpigmented scaly
plaque.
• Both coffee and Cox-2 inhibitors may prevent non-melanoma
skin cancer like SCC.

“Skin cancer, if caught early, can be treated fairly
straightforwardly, and it’s not that big a deal. But once skin
cancer has spread, effectively nothing works. So it’s particularly
important to catch skin cancer early.”

~ Dr. Darrell Rigel, American dermatologist

63
Question 1 of 3
What is your diagnosis?

A. Impetigo
B. Molluscum contagiosum
C. Juvenile xanthogranuloma
D. Milia
P APULES ON THE ABDOMEN
A 5 year-old boy was brought into clinic by his mother
because of a concern regarding papules that were
developing on his abdomen, chest and upper arms.
These papules were mildly pruritic, and had been
present for 2 months and new papules were still
appearing. There were no sick contacts, and the child
felt well otherwise.

E. Common Warts

Check Answer

Diagnosis & Information
64
typically occur on the chest, arms, trunk, legs, and face. Mucous
membrane involvement is rare, and the palmoplantar area is
spared. Patients with a history of atopic dermatitis are more prone
to MC and may develop a large number of lesions. Approximately
10% of all patients will develop eczema around the lesions. In
adults, MC is most commonly a sexually transmitted disease and
presents as a few scattered lesions often limited to the perineum,
genitalia, inner thighs, lower abdomen, or buttocks. MC in healthy
children and adults is usually a self-limited disease, but may
persist for several months and even up to a few years. Widespread,
persistent, and atypical MC may occur in patients who are
significantly immunocompromised or have acquired
immunodeficiency syndrome (AIDS) with low CD4 T-lymphocyte
counts.

Diagnosis: Molluscum contagiosum
Molluscum contagiosum (MC) is a viral infection of the skin
caused by a DNA poxvirus called the molluscum contagiosum
virus (MCV), which is a benign and generally self-limited viral
infection. Skin lesions typically consist of multiple dome-shaped,
pink to skin-colored papules 2-6 mm in diameter of which some
lesions show the classic feature of umbilication. MC is usually
asymptomatic, although individual lesions can occasionally be
tender or pruritic.
MC is encountered most commonly in children who become
infected through direct skin-to-skin contact or indirect skin
contact with fomites, as well as by auto-inoculation. Lesions

Diagnosis is usually clinical and based on the distinctive central
umbilication of the dome-shaped papule. If diagnosis is uncertain,
papules can be biopsied and sent for histopathologic
identification. Adult patients should be questioned about sexual
history and where appropriate, evaluated for other concomitant
STDs. Always consider testing for HIV infection in patients with
large or facial MC lesions.
Patients and their families should be educated as to the benign
and self-limited nature of this condition and that treatment is not
a necessity. For the most part, the main concern is temporary
adverse cosmetic results and embarrassment. Most lesions resolve
with no permanent residual skin defect; however, occasional
lesions may produce a slightly depressed scar, especially if
excoriated. Although treatment is not required, it can help reduce
autoinoculation or transmission to close contacts and improve
clinical appearance. More than one treatment session is frequently
required.
65
In healthy children, a major goal is to limit physical discomfort.
Often, no medical intervention or minor direct trauma is
appropriate. Cryotherapy or curettage (can apply topical
anesthetic cream pre-procedure) are very effective therapies for
older children. For younger children, Cantharidin applied
carefully to the lesions and washed off after 15 minutes to 2 hours
can be effective. Similarly, tretinoin/vitamin-A acid applied
topically only to the lesions (e.g. with a toothpick) can be used.
Another treatment option to consider, with variable benefit, is
topical imiquimod (e.g. Aldara™, Zyclara™), a topical immune
response modifier and a potent inducer of anti-viral interferons.

“What perhaps should receive more attention is the effect of the
treatment on the virus.” ~ Luc Montagnier, French virologist
and 2008 Nobel Prize Winner (Discovery of HIV)

In adults who are more motivated to have their lesions treated,
cryotherapy or curettage of individual lesions is effective and well
tolerated. In immunocompromised individuals, MC can be
extensive and difficult to treat.
The prognosis of patients with MC is generally excellent because
the disease is benign and self-limited. In healthy patients, 1-4
treatments are usually effective to see complete resolution of the
lesions.

Clinical Pearl
• Be careful when treating self-limited conditions such as
molluscum or warts in individuals with more pigmented skin
as the treatments can result in hyper or hypo-pigmentation
which often bother the patient even more than the original
problem and can have medico-legal ramifications.

66
Question 1 of 3
What is his diagnosis?

A. Linear IgA disease
B. Herpes simplex
C. Herpes zoster
D. Dermatitis herpetiformis
P AINFUL , VESICULAR RASH
A 30 year-old male presents with a painful vesicular
rash to his right lateral flank region. He has been
experiencing generalized malaise and myalgias to the
affected area for the past 2 days.

E. None of the above

Check Answer

Diagnosis & Information
67
Diagnosis: Herpes zoster
Herpes zoster (shingles) is an acute, painful dermatomal
dermatitis that affects approximately 10% to 20% of adults, often
in the presence of immunosuppression. Herpes zoster is a viral
infection caused by the same virus (varicella) that causes chicken
pox. During the course of varicella, the virus travels from the skin
and mucosal surfaces to the sensory ganglia, where it lies dormant
for a patient’s lifetime. Reactivation often follows
immunosuppression, illness, emotional stress, trauma, and
irradiation or surgical manipulation of the spine, producing a
dermatomal dermatitis.

Herpes zoster is primarily a disease of adults and typically begins
with pain and paresthesia in a dermatomal or bandlike pattern
followed by grouped vesicles within the dermatome several days
later. Occasionally, fever and malaise occur. The thoracic area
accounts for more than half of all reported cases. When zoster
involves the tip and side of the nose (cranial nerve V) nasociliary
nerve involvement can occur (30%-40%). Great care is needed if
the blisters involve the eye because permanent eye damage can
result, and so an ophthalmology consultation should be strongly
considered. Most patients with zoster do well with only
symptomatic treatment, but postherpetic neuralgia is common in
the elderly. Disseminated zoster is uncommon and occurs
primarily in immunocompromised patients. Since 2008, a live,
injectable, attenuated herpes zoster vaccine (Zostavax™) has been
authorized for use in Canada for the prevention of herpes zoster
(shingles) infection in adults aged 60 years and older. Since its
inception, the vaccine has greatly reduced the number of cases
of postherpetic neuralgia and reduced the severity and duration of
pain and discomfort associated with shingles.
Diagnosis is by clinical presentation, viral culture, or direct
fluorescent antibody from a skin biopsy. Prompt treatment
reduces the extent and duration of symptoms as well as the risk of
chronic sequelae. Antiviral agents (such as acyclovir, valacyclovir
and famciclovir) promote healing of cutaneous lesions and
prevent or reduce the severity of postherpetic neuralgia. The
recommended adult dose of valacyclovir (Valtrex®) to treat
shingles is 1000 mg tid for 7 days. The treatment should be
initiated within 72 hours of the onset of the rash for better
management of clinical sequelae. Narcotic and non-narcotic
topical and oral analgesics, anti-convulsants, and tricyclic

68
antidepressants can be used for acute zoster-associated
neuralgias. The use of oral corticosteroid therapy is controversial.
Botulinum toxin (Botox®) is also effective for management of
pain caused by postherpetic neuralgia. The analgesic effects are
not dependant on changes in muscle tone. Clinical benefit with
Botox® is seen for at least 14 weeks after it is given. Local
anesthetics applied directly to the skin of the painful area affected
by postherpetic neuralgia are also effective. Lidocaine
(Xylocaine®) is available in a patch that has been approved for
use specifically in postherpetic neuralgia. It can be used to the
affected area every 4-12 hours as needed. Topical local anesthetics
stay in the skin and therefore do not cause problems such as
drowsiness or constipation. Capsaicin (Zostrix®) cream may be
somewhat effective and is available over the counter. It can be
applied to the affected area 3-5 times daily. However, patients
often find that it causes a burning pain during application.

Clinical Pearl
• Be careful when treating self-limited conditions such as
molluscum or warts in individuals with more pigmented skin
as the treatments can result in hyper or hypo-pigmentation
which often bother the patient even more than the original
problem and can have medico-legal ramifications.

“In making a dermatologic diagnosis, look for the distinctive
face of the disease, not its featureless arms, legs and trunk.”
~ Drs. Walter & Dorinda Shelley, American Dermatologists and
authors of Advanced Dermatologic Diagnosis

69
Question 1 of 3
What is the diagnosis?

A. Inverse psoriasis
B. Atopic dermatitis
C. Mycosis fungoides
D. Contact dermatitis
I TCHY , ROUGH PATCHES ON LEGS
A 41 year-old woman presents with a 6 month history of
pruritic lesions affecting the popliteal fossae. She
notices the lesions are more painful and itchy in the
winter. She has tried some over the counter
moisturizers that have only slightly helped. She had
asthma and mild exzema as a child but is otherwise
healthy and takes no medications. She has an older
sister with a history of eczema.

E. Nummular eczema

Check Answer

Diagnosis & Information
70
Derm book
Derm book
Derm book
Derm book
Derm book
Derm book
Derm book
Derm book
Derm book
Derm book
Derm book
Derm book
Derm book
Derm book
Derm book
Derm book
Derm book
Derm book
Derm book
Derm book
Derm book
Derm book
Derm book
Derm book
Derm book
Derm book
Derm book
Derm book
Derm book
Derm book
Derm book
Derm book
Derm book
Derm book
Derm book

Mais conteúdo relacionado

Mais procurados

Dr. Patrick Treacy lectures on IPL (Dublin)
Dr. Patrick Treacy lectures on IPL (Dublin) Dr. Patrick Treacy lectures on IPL (Dublin)
Dr. Patrick Treacy lectures on IPL (Dublin) Dr. Patrick J. Treacy
 
Approach to photodermatoses
Approach to photodermatosesApproach to photodermatoses
Approach to photodermatosesDrYusraShabbir
 
Ablative & Nonablative Lasers for Face Rejuvenation
Ablative & Nonablative Lasers for Face RejuvenationAblative & Nonablative Lasers for Face Rejuvenation
Ablative & Nonablative Lasers for Face RejuvenationXristoforos Tzermias
 
Dermatological manifestation of systemic diseases
Dermatological manifestation of systemic diseasesDermatological manifestation of systemic diseases
Dermatological manifestation of systemic diseasesShivshankar Badole
 
Fractional co2 laser
Fractional co2 laser Fractional co2 laser
Fractional co2 laser Mindy Ma
 
Introduction To Dermatopathology
Introduction To DermatopathologyIntroduction To Dermatopathology
Introduction To DermatopathologyIbrahim Farag
 
BASICS OF LASER AND IT'S USE IN DERMATOLOGY
BASICS OF LASER AND IT'S USE IN DERMATOLOGYBASICS OF LASER AND IT'S USE IN DERMATOLOGY
BASICS OF LASER AND IT'S USE IN DERMATOLOGYRohit Singh
 
Wood’s light in dermatology
Wood’s light in dermatologyWood’s light in dermatology
Wood’s light in dermatologyAzza Samy
 
Dermatological history and examination
Dermatological history and examinationDermatological history and examination
Dermatological history and examinationFilmNatapol
 
differentials of papules on face
differentials of papules on facedifferentials of papules on face
differentials of papules on faceMikhin Thomas
 
Introduction to laser dermatology 2
Introduction to laser dermatology 2Introduction to laser dermatology 2
Introduction to laser dermatology 2Islam Noaman
 

Mais procurados (20)

Dr. Patrick Treacy lectures on IPL (Dublin)
Dr. Patrick Treacy lectures on IPL (Dublin) Dr. Patrick Treacy lectures on IPL (Dublin)
Dr. Patrick Treacy lectures on IPL (Dublin)
 
Erythroderma
ErythrodermaErythroderma
Erythroderma
 
Standardizing Outcome Measures in Alopecia Areata
Standardizing Outcome Measures in Alopecia AreataStandardizing Outcome Measures in Alopecia Areata
Standardizing Outcome Measures in Alopecia Areata
 
Approach to photodermatoses
Approach to photodermatosesApproach to photodermatoses
Approach to photodermatoses
 
Ichthyosis
IchthyosisIchthyosis
Ichthyosis
 
Alopecia Areata
Alopecia AreataAlopecia Areata
Alopecia Areata
 
Understanding eczema
Understanding eczemaUnderstanding eczema
Understanding eczema
 
HAIR Disorders.pptx
HAIR Disorders.pptxHAIR Disorders.pptx
HAIR Disorders.pptx
 
Telogen Effluvium
Telogen EffluviumTelogen Effluvium
Telogen Effluvium
 
Porokeratosis
PorokeratosisPorokeratosis
Porokeratosis
 
Ablative & Nonablative Lasers for Face Rejuvenation
Ablative & Nonablative Lasers for Face RejuvenationAblative & Nonablative Lasers for Face Rejuvenation
Ablative & Nonablative Lasers for Face Rejuvenation
 
Dermatological manifestation of systemic diseases
Dermatological manifestation of systemic diseasesDermatological manifestation of systemic diseases
Dermatological manifestation of systemic diseases
 
Fractional co2 laser
Fractional co2 laser Fractional co2 laser
Fractional co2 laser
 
Introduction To Dermatopathology
Introduction To DermatopathologyIntroduction To Dermatopathology
Introduction To Dermatopathology
 
BASICS OF LASER AND IT'S USE IN DERMATOLOGY
BASICS OF LASER AND IT'S USE IN DERMATOLOGYBASICS OF LASER AND IT'S USE IN DERMATOLOGY
BASICS OF LASER AND IT'S USE IN DERMATOLOGY
 
Wood’s light in dermatology
Wood’s light in dermatologyWood’s light in dermatology
Wood’s light in dermatology
 
Cutaneous amyloidosis
Cutaneous amyloidosisCutaneous amyloidosis
Cutaneous amyloidosis
 
Dermatological history and examination
Dermatological history and examinationDermatological history and examination
Dermatological history and examination
 
differentials of papules on face
differentials of papules on facedifferentials of papules on face
differentials of papules on face
 
Introduction to laser dermatology 2
Introduction to laser dermatology 2Introduction to laser dermatology 2
Introduction to laser dermatology 2
 

Destaque

Comparative evaluation of pricking pain response, behavioural response, heart...
Comparative evaluation of pricking pain response, behavioural response, heart...Comparative evaluation of pricking pain response, behavioural response, heart...
Comparative evaluation of pricking pain response, behavioural response, heart...iosrjce
 
Common skin problems
Common skin problemsCommon skin problems
Common skin problemssajith8523
 
Skin And Soft Tissue Infections
Skin And Soft Tissue InfectionsSkin And Soft Tissue Infections
Skin And Soft Tissue InfectionsMiami Dade
 
Ca breast, diagnosis, clinical examination and diagnostic workup
Ca breast, diagnosis, clinical examination and diagnostic workup Ca breast, diagnosis, clinical examination and diagnostic workup
Ca breast, diagnosis, clinical examination and diagnostic workup Satyajeet Rath
 
Diabetes in bangla dr md toufiqur rahman nicvd cardiologist final
Diabetes in bangla dr md toufiqur rahman nicvd cardiologist finalDiabetes in bangla dr md toufiqur rahman nicvd cardiologist final
Diabetes in bangla dr md toufiqur rahman nicvd cardiologist finalPROFESSOR DR. MD. TOUFIQUR RAHMAN
 
Dermatology osce slides
Dermatology osce slidesDermatology osce slides
Dermatology osce slidesHabrol Afzam
 
Common Skin Diseases
Common Skin DiseasesCommon Skin Diseases
Common Skin Diseasesdoctorshazly
 
Psoriasis-The best Presentation
Psoriasis-The best PresentationPsoriasis-The best Presentation
Psoriasis-The best PresentationDr.Shahidul Islam
 
Taller slideshare y prezi sofia mantillam
Taller slideshare y prezi sofia mantillamTaller slideshare y prezi sofia mantillam
Taller slideshare y prezi sofia mantillamSofia Mantilla
 
Elsabe de Beer cv
Elsabe de Beer cvElsabe de Beer cv
Elsabe de Beer cvDe Elsabe
 

Destaque (16)

Quest Ver2
Quest Ver2Quest Ver2
Quest Ver2
 
Comparative evaluation of pricking pain response, behavioural response, heart...
Comparative evaluation of pricking pain response, behavioural response, heart...Comparative evaluation of pricking pain response, behavioural response, heart...
Comparative evaluation of pricking pain response, behavioural response, heart...
 
Derma.
Derma.Derma.
Derma.
 
Common skin problems
Common skin problemsCommon skin problems
Common skin problems
 
Skin And Soft Tissue Infections
Skin And Soft Tissue InfectionsSkin And Soft Tissue Infections
Skin And Soft Tissue Infections
 
Ca breast, diagnosis, clinical examination and diagnostic workup
Ca breast, diagnosis, clinical examination and diagnostic workup Ca breast, diagnosis, clinical examination and diagnostic workup
Ca breast, diagnosis, clinical examination and diagnostic workup
 
Diabetes in bangla dr md toufiqur rahman nicvd cardiologist final
Diabetes in bangla dr md toufiqur rahman nicvd cardiologist finalDiabetes in bangla dr md toufiqur rahman nicvd cardiologist final
Diabetes in bangla dr md toufiqur rahman nicvd cardiologist final
 
Mrcp 2 dermatology
Mrcp 2 dermatologyMrcp 2 dermatology
Mrcp 2 dermatology
 
Dermatology made easy
Dermatology made easyDermatology made easy
Dermatology made easy
 
Dermatology osce slides
Dermatology osce slidesDermatology osce slides
Dermatology osce slides
 
Common Skin Diseases
Common Skin DiseasesCommon Skin Diseases
Common Skin Diseases
 
Psoriasis-The best Presentation
Psoriasis-The best PresentationPsoriasis-The best Presentation
Psoriasis-The best Presentation
 
Diarreas trabajo de computo
Diarreas trabajo de computoDiarreas trabajo de computo
Diarreas trabajo de computo
 
Taller slideshare y prezi sofia mantillam
Taller slideshare y prezi sofia mantillamTaller slideshare y prezi sofia mantillam
Taller slideshare y prezi sofia mantillam
 
Elsabe de Beer cv
Elsabe de Beer cvElsabe de Beer cv
Elsabe de Beer cv
 
thomas cv
thomas cvthomas cv
thomas cv
 

Semelhante a Derm book

Abstract Workshop On Psycho Oncology Alleviation.7[1]
Abstract  Workshop On Psycho Oncology  Alleviation.7[1]Abstract  Workshop On Psycho Oncology  Alleviation.7[1]
Abstract Workshop On Psycho Oncology Alleviation.7[1]Al-Sadeel Society
 
1)What is MWLs service concept, and what is your evaluation of it.docx
1)What is MWLs service concept, and what is your evaluation of it.docx1)What is MWLs service concept, and what is your evaluation of it.docx
1)What is MWLs service concept, and what is your evaluation of it.docxSONU61709
 
Cultural competence in dermatology
Cultural competence in dermatologyCultural competence in dermatology
Cultural competence in dermatologyLindsey Lawrence
 
How To Write A Medical Case Report - BMJ
How To Write A Medical Case Report - BMJHow To Write A Medical Case Report - BMJ
How To Write A Medical Case Report - BMJAlana Cartwright
 
Conversations About Financial Issues in Routine Oncology Practices: A Multice...
Conversations About Financial Issues in Routine Oncology Practices: A Multice...Conversations About Financial Issues in Routine Oncology Practices: A Multice...
Conversations About Financial Issues in Routine Oncology Practices: A Multice...Melissa Paige
 
#35773 Topic PROBLEM STATEMENT AND LIT REVIEW FOR END OF LIFE CAR.docx
#35773 Topic PROBLEM STATEMENT AND LIT REVIEW FOR END OF LIFE CAR.docx#35773 Topic PROBLEM STATEMENT AND LIT REVIEW FOR END OF LIFE CAR.docx
#35773 Topic PROBLEM STATEMENT AND LIT REVIEW FOR END OF LIFE CAR.docxAASTHA76
 
Derm handbook for medical students and junior doctors 2010
Derm handbook for medical students and junior doctors 2010Derm handbook for medical students and junior doctors 2010
Derm handbook for medical students and junior doctors 2010Derma202
 
Best practice-antiseptic-antimicrobial
Best practice-antiseptic-antimicrobialBest practice-antiseptic-antimicrobial
Best practice-antiseptic-antimicrobialGNEAUPP.
 
Clinical_Handbook_on_Canine_Dermatology.pdf
Clinical_Handbook_on_Canine_Dermatology.pdfClinical_Handbook_on_Canine_Dermatology.pdf
Clinical_Handbook_on_Canine_Dermatology.pdfSenseiRobertAlemn
 
Challenged Conceptions - Environmental Chemicals & Fertility
Challenged Conceptions - Environmental Chemicals & Fertility Challenged Conceptions - Environmental Chemicals & Fertility
Challenged Conceptions - Environmental Chemicals & Fertility v2zq
 
Evidence based periodontology
Evidence based periodontologyEvidence based periodontology
Evidence based periodontologySumalatha Appam
 
Introduction to Evidence Based Psychiatry.pptx
Introduction to Evidence Based Psychiatry.pptxIntroduction to Evidence Based Psychiatry.pptx
Introduction to Evidence Based Psychiatry.pptxFranklinAlier
 
The Role of the Pharmacist in Patient Care (Book Review)
The Role of the Pharmacist in Patient Care (Book Review)The Role of the Pharmacist in Patient Care (Book Review)
The Role of the Pharmacist in Patient Care (Book Review)clinicsoncology
 
The Role of the Pharmacist in Patient Care (Book Review)
The Role of the Pharmacist in Patient Care (Book Review)The Role of the Pharmacist in Patient Care (Book Review)
The Role of the Pharmacist in Patient Care (Book Review)pateldrona
 
The Role of the Pharmacist in Patient Care (Book Review)
The Role of the Pharmacist in Patient Care (Book Review)The Role of the Pharmacist in Patient Care (Book Review)
The Role of the Pharmacist in Patient Care (Book Review)komalicarol
 

Semelhante a Derm book (20)

Abstract Workshop On Psycho Oncology Alleviation.7[1]
Abstract  Workshop On Psycho Oncology  Alleviation.7[1]Abstract  Workshop On Psycho Oncology  Alleviation.7[1]
Abstract Workshop On Psycho Oncology Alleviation.7[1]
 
1)What is MWLs service concept, and what is your evaluation of it.docx
1)What is MWLs service concept, and what is your evaluation of it.docx1)What is MWLs service concept, and what is your evaluation of it.docx
1)What is MWLs service concept, and what is your evaluation of it.docx
 
Cultural competence in dermatology
Cultural competence in dermatologyCultural competence in dermatology
Cultural competence in dermatology
 
2020 Knowledge Translation Student Award Panel - Courtney Primeau, PhD, Unive...
2020 Knowledge Translation Student Award Panel - Courtney Primeau, PhD, Unive...2020 Knowledge Translation Student Award Panel - Courtney Primeau, PhD, Unive...
2020 Knowledge Translation Student Award Panel - Courtney Primeau, PhD, Unive...
 
How To Write A Medical Case Report - BMJ
How To Write A Medical Case Report - BMJHow To Write A Medical Case Report - BMJ
How To Write A Medical Case Report - BMJ
 
FINAL PAPER 432
FINAL PAPER 432FINAL PAPER 432
FINAL PAPER 432
 
Conversations About Financial Issues in Routine Oncology Practices: A Multice...
Conversations About Financial Issues in Routine Oncology Practices: A Multice...Conversations About Financial Issues in Routine Oncology Practices: A Multice...
Conversations About Financial Issues in Routine Oncology Practices: A Multice...
 
Clinical Cases in Emergency Medicine - prelims
Clinical Cases in Emergency Medicine - prelimsClinical Cases in Emergency Medicine - prelims
Clinical Cases in Emergency Medicine - prelims
 
#35773 Topic PROBLEM STATEMENT AND LIT REVIEW FOR END OF LIFE CAR.docx
#35773 Topic PROBLEM STATEMENT AND LIT REVIEW FOR END OF LIFE CAR.docx#35773 Topic PROBLEM STATEMENT AND LIT REVIEW FOR END OF LIFE CAR.docx
#35773 Topic PROBLEM STATEMENT AND LIT REVIEW FOR END OF LIFE CAR.docx
 
Derm handbook for medical students and junior doctors 2010
Derm handbook for medical students and junior doctors 2010Derm handbook for medical students and junior doctors 2010
Derm handbook for medical students and junior doctors 2010
 
WholePersonCare Kp
WholePersonCare  KpWholePersonCare  Kp
WholePersonCare Kp
 
Best practice-antiseptic-antimicrobial
Best practice-antiseptic-antimicrobialBest practice-antiseptic-antimicrobial
Best practice-antiseptic-antimicrobial
 
Clinical_Handbook_on_Canine_Dermatology.pdf
Clinical_Handbook_on_Canine_Dermatology.pdfClinical_Handbook_on_Canine_Dermatology.pdf
Clinical_Handbook_on_Canine_Dermatology.pdf
 
Challenged Conceptions - Environmental Chemicals & Fertility
Challenged Conceptions - Environmental Chemicals & Fertility Challenged Conceptions - Environmental Chemicals & Fertility
Challenged Conceptions - Environmental Chemicals & Fertility
 
Evidence based periodontology
Evidence based periodontologyEvidence based periodontology
Evidence based periodontology
 
Making a case for medication reconciliation in primary care
Making a case for medication reconciliation in primary careMaking a case for medication reconciliation in primary care
Making a case for medication reconciliation in primary care
 
Introduction to Evidence Based Psychiatry.pptx
Introduction to Evidence Based Psychiatry.pptxIntroduction to Evidence Based Psychiatry.pptx
Introduction to Evidence Based Psychiatry.pptx
 
The Role of the Pharmacist in Patient Care (Book Review)
The Role of the Pharmacist in Patient Care (Book Review)The Role of the Pharmacist in Patient Care (Book Review)
The Role of the Pharmacist in Patient Care (Book Review)
 
The Role of the Pharmacist in Patient Care (Book Review)
The Role of the Pharmacist in Patient Care (Book Review)The Role of the Pharmacist in Patient Care (Book Review)
The Role of the Pharmacist in Patient Care (Book Review)
 
The Role of the Pharmacist in Patient Care (Book Review)
The Role of the Pharmacist in Patient Care (Book Review)The Role of the Pharmacist in Patient Care (Book Review)
The Role of the Pharmacist in Patient Care (Book Review)
 

Mais de Ihsaan Peer

Pef reference and chart
Pef reference and chartPef reference and chart
Pef reference and chartIhsaan Peer
 
Gold slideset cop_djan14
Gold slideset cop_djan14Gold slideset cop_djan14
Gold slideset cop_djan14Ihsaan Peer
 
Feb 2014 allergy b clinical and aqhi
Feb 2014 allergy b clinical and aqhiFeb 2014 allergy b clinical and aqhi
Feb 2014 allergy b clinical and aqhiIhsaan Peer
 
Feb 2014 allergy a physiology
Feb 2014 allergy a physiologyFeb 2014 allergy a physiology
Feb 2014 allergy a physiologyIhsaan Peer
 
Chest painassessment
Chest painassessmentChest painassessment
Chest painassessmentIhsaan Peer
 
A1 at review can fam phy(1)
A1 at review can fam phy(1)A1 at review can fam phy(1)
A1 at review can fam phy(1)Ihsaan Peer
 
2012 cts guidline_alpha-1
2012 cts guidline_alpha-12012 cts guidline_alpha-1
2012 cts guidline_alpha-1Ihsaan Peer
 
265 wa uninsured services and billing
265 wa uninsured services and billing265 wa uninsured services and billing
265 wa uninsured services and billingIhsaan Peer
 
Pmh presentation
Pmh presentationPmh presentation
Pmh presentationIhsaan Peer
 
Sat 0810-gallagher-end-of-life-care- -park
Sat 0810-gallagher-end-of-life-care- -parkSat 0810-gallagher-end-of-life-care- -park
Sat 0810-gallagher-end-of-life-care- -parkIhsaan Peer
 
Sat 0810-smith-case-for-legalizing-medically-assisted-dying-in-canada- -park
Sat 0810-smith-case-for-legalizing-medically-assisted-dying-in-canada- -parkSat 0810-smith-case-for-legalizing-medically-assisted-dying-in-canada- -park
Sat 0810-smith-case-for-legalizing-medically-assisted-dying-in-canada- -parkIhsaan Peer
 
Sat 0855-hepatitis-c-update- -park
Sat 0855-hepatitis-c-update- -parkSat 0855-hepatitis-c-update- -park
Sat 0855-hepatitis-c-update- -parkIhsaan Peer
 
Sat 1025-hair-management-too-much-too-little- -park
Sat 1025-hair-management-too-much-too-little- -parkSat 1025-hair-management-too-much-too-little- -park
Sat 1025-hair-management-too-much-too-little- -parkIhsaan Peer
 
Sat 1110-food-allergies- -seasons
Sat 1110-food-allergies- -seasonsSat 1110-food-allergies- -seasons
Sat 1110-food-allergies- -seasonsIhsaan Peer
 
Sat 1110-health-e apps---garibaldi
Sat 1110-health-e apps---garibaldiSat 1110-health-e apps---garibaldi
Sat 1110-health-e apps---garibaldiIhsaan Peer
 
Sat 1420-infertility- -garibaldi
Sat 1420-infertility- -garibaldiSat 1420-infertility- -garibaldi
Sat 1420-infertility- -garibaldiIhsaan Peer
 
Sat 1420-lower-back-exam- -park
Sat 1420-lower-back-exam- -parkSat 1420-lower-back-exam- -park
Sat 1420-lower-back-exam- -parkIhsaan Peer
 
Sat 1420-prescribing-exercise- -arbutus
Sat 1420-prescribing-exercise- -arbutusSat 1420-prescribing-exercise- -arbutus
Sat 1420-prescribing-exercise- -arbutusIhsaan Peer
 
Sat 1420-thyrotoxicosis- -seasons
Sat 1420-thyrotoxicosis- -seasonsSat 1420-thyrotoxicosis- -seasons
Sat 1420-thyrotoxicosis- -seasonsIhsaan Peer
 

Mais de Ihsaan Peer (20)

Vte 2014
Vte 2014Vte 2014
Vte 2014
 
Pef reference and chart
Pef reference and chartPef reference and chart
Pef reference and chart
 
Gold slideset cop_djan14
Gold slideset cop_djan14Gold slideset cop_djan14
Gold slideset cop_djan14
 
Feb 2014 allergy b clinical and aqhi
Feb 2014 allergy b clinical and aqhiFeb 2014 allergy b clinical and aqhi
Feb 2014 allergy b clinical and aqhi
 
Feb 2014 allergy a physiology
Feb 2014 allergy a physiologyFeb 2014 allergy a physiology
Feb 2014 allergy a physiology
 
Chest painassessment
Chest painassessmentChest painassessment
Chest painassessment
 
A1 at review can fam phy(1)
A1 at review can fam phy(1)A1 at review can fam phy(1)
A1 at review can fam phy(1)
 
2012 cts guidline_alpha-1
2012 cts guidline_alpha-12012 cts guidline_alpha-1
2012 cts guidline_alpha-1
 
265 wa uninsured services and billing
265 wa uninsured services and billing265 wa uninsured services and billing
265 wa uninsured services and billing
 
Pmh presentation
Pmh presentationPmh presentation
Pmh presentation
 
Sat 0810-gallagher-end-of-life-care- -park
Sat 0810-gallagher-end-of-life-care- -parkSat 0810-gallagher-end-of-life-care- -park
Sat 0810-gallagher-end-of-life-care- -park
 
Sat 0810-smith-case-for-legalizing-medically-assisted-dying-in-canada- -park
Sat 0810-smith-case-for-legalizing-medically-assisted-dying-in-canada- -parkSat 0810-smith-case-for-legalizing-medically-assisted-dying-in-canada- -park
Sat 0810-smith-case-for-legalizing-medically-assisted-dying-in-canada- -park
 
Sat 0855-hepatitis-c-update- -park
Sat 0855-hepatitis-c-update- -parkSat 0855-hepatitis-c-update- -park
Sat 0855-hepatitis-c-update- -park
 
Sat 1025-hair-management-too-much-too-little- -park
Sat 1025-hair-management-too-much-too-little- -parkSat 1025-hair-management-too-much-too-little- -park
Sat 1025-hair-management-too-much-too-little- -park
 
Sat 1110-food-allergies- -seasons
Sat 1110-food-allergies- -seasonsSat 1110-food-allergies- -seasons
Sat 1110-food-allergies- -seasons
 
Sat 1110-health-e apps---garibaldi
Sat 1110-health-e apps---garibaldiSat 1110-health-e apps---garibaldi
Sat 1110-health-e apps---garibaldi
 
Sat 1420-infertility- -garibaldi
Sat 1420-infertility- -garibaldiSat 1420-infertility- -garibaldi
Sat 1420-infertility- -garibaldi
 
Sat 1420-lower-back-exam- -park
Sat 1420-lower-back-exam- -parkSat 1420-lower-back-exam- -park
Sat 1420-lower-back-exam- -park
 
Sat 1420-prescribing-exercise- -arbutus
Sat 1420-prescribing-exercise- -arbutusSat 1420-prescribing-exercise- -arbutus
Sat 1420-prescribing-exercise- -arbutus
 
Sat 1420-thyrotoxicosis- -seasons
Sat 1420-thyrotoxicosis- -seasonsSat 1420-thyrotoxicosis- -seasons
Sat 1420-thyrotoxicosis- -seasons
 

Último

VIP Call Girls Lucknow Nandini 7001305949 Independent Escort Service Lucknow
VIP Call Girls Lucknow Nandini 7001305949 Independent Escort Service LucknowVIP Call Girls Lucknow Nandini 7001305949 Independent Escort Service Lucknow
VIP Call Girls Lucknow Nandini 7001305949 Independent Escort Service Lucknownarwatsonia7
 
call girls in munirka DELHI 🔝 >༒9540349809 🔝 genuine Escort Service 🔝✔️✔️
call girls in munirka  DELHI 🔝 >༒9540349809 🔝 genuine Escort Service 🔝✔️✔️call girls in munirka  DELHI 🔝 >༒9540349809 🔝 genuine Escort Service 🔝✔️✔️
call girls in munirka DELHI 🔝 >༒9540349809 🔝 genuine Escort Service 🔝✔️✔️saminamagar
 
Call Girls Jp Nagar Just Call 7001305949 Top Class Call Girl Service Available
Call Girls Jp Nagar Just Call 7001305949 Top Class Call Girl Service AvailableCall Girls Jp Nagar Just Call 7001305949 Top Class Call Girl Service Available
Call Girls Jp Nagar Just Call 7001305949 Top Class Call Girl Service Availablenarwatsonia7
 
Hematology and Immunology - Leukocytes Functions
Hematology and Immunology - Leukocytes FunctionsHematology and Immunology - Leukocytes Functions
Hematology and Immunology - Leukocytes FunctionsMedicoseAcademics
 
Call Girls Hebbal Just Call 7001305949 Top Class Call Girl Service Available
Call Girls Hebbal Just Call 7001305949 Top Class Call Girl Service AvailableCall Girls Hebbal Just Call 7001305949 Top Class Call Girl Service Available
Call Girls Hebbal Just Call 7001305949 Top Class Call Girl Service Availablenarwatsonia7
 
Call Girls Hsr Layout Just Call 7001305949 Top Class Call Girl Service Available
Call Girls Hsr Layout Just Call 7001305949 Top Class Call Girl Service AvailableCall Girls Hsr Layout Just Call 7001305949 Top Class Call Girl Service Available
Call Girls Hsr Layout Just Call 7001305949 Top Class Call Girl Service Availablenarwatsonia7
 
Russian Call Girl Brookfield - 7001305949 Escorts Service 50% Off with Cash O...
Russian Call Girl Brookfield - 7001305949 Escorts Service 50% Off with Cash O...Russian Call Girl Brookfield - 7001305949 Escorts Service 50% Off with Cash O...
Russian Call Girl Brookfield - 7001305949 Escorts Service 50% Off with Cash O...narwatsonia7
 
Kolkata Call Girls Services 9907093804 @24x7 High Class Babes Here Call Now
Kolkata Call Girls Services 9907093804 @24x7 High Class Babes Here Call NowKolkata Call Girls Services 9907093804 @24x7 High Class Babes Here Call Now
Kolkata Call Girls Services 9907093804 @24x7 High Class Babes Here Call NowNehru place Escorts
 
Call Girls Service Nandiambakkam | 7001305949 At Low Cost Cash Payment Booking
Call Girls Service Nandiambakkam | 7001305949 At Low Cost Cash Payment BookingCall Girls Service Nandiambakkam | 7001305949 At Low Cost Cash Payment Booking
Call Girls Service Nandiambakkam | 7001305949 At Low Cost Cash Payment BookingNehru place Escorts
 
Call Girl Nagpur Sia 7001305949 Independent Escort Service Nagpur
Call Girl Nagpur Sia 7001305949 Independent Escort Service NagpurCall Girl Nagpur Sia 7001305949 Independent Escort Service Nagpur
Call Girl Nagpur Sia 7001305949 Independent Escort Service NagpurRiya Pathan
 
Housewife Call Girls Bangalore - Call 7001305949 Rs-3500 with A/C Room Cash o...
Housewife Call Girls Bangalore - Call 7001305949 Rs-3500 with A/C Room Cash o...Housewife Call Girls Bangalore - Call 7001305949 Rs-3500 with A/C Room Cash o...
Housewife Call Girls Bangalore - Call 7001305949 Rs-3500 with A/C Room Cash o...narwatsonia7
 
Call Girls Frazer Town Just Call 7001305949 Top Class Call Girl Service Avail...
Call Girls Frazer Town Just Call 7001305949 Top Class Call Girl Service Avail...Call Girls Frazer Town Just Call 7001305949 Top Class Call Girl Service Avail...
Call Girls Frazer Town Just Call 7001305949 Top Class Call Girl Service Avail...narwatsonia7
 
See the 2,456 pharmacies on the National E-Pharmacy Platform
See the 2,456 pharmacies on the National E-Pharmacy PlatformSee the 2,456 pharmacies on the National E-Pharmacy Platform
See the 2,456 pharmacies on the National E-Pharmacy PlatformKweku Zurek
 
Russian Call Girls Chickpet - 7001305949 Booking and charges genuine rate for...
Russian Call Girls Chickpet - 7001305949 Booking and charges genuine rate for...Russian Call Girls Chickpet - 7001305949 Booking and charges genuine rate for...
Russian Call Girls Chickpet - 7001305949 Booking and charges genuine rate for...narwatsonia7
 
Call Girls In Andheri East Call 9920874524 Book Hot And Sexy Girls
Call Girls In Andheri East Call 9920874524 Book Hot And Sexy GirlsCall Girls In Andheri East Call 9920874524 Book Hot And Sexy Girls
Call Girls In Andheri East Call 9920874524 Book Hot And Sexy Girlsnehamumbai
 
Glomerular Filtration and determinants of glomerular filtration .pptx
Glomerular Filtration and  determinants of glomerular filtration .pptxGlomerular Filtration and  determinants of glomerular filtration .pptx
Glomerular Filtration and determinants of glomerular filtration .pptxDr.Nusrat Tariq
 
Call Girls Kanakapura Road Just Call 7001305949 Top Class Call Girl Service A...
Call Girls Kanakapura Road Just Call 7001305949 Top Class Call Girl Service A...Call Girls Kanakapura Road Just Call 7001305949 Top Class Call Girl Service A...
Call Girls Kanakapura Road Just Call 7001305949 Top Class Call Girl Service A...narwatsonia7
 
Book Call Girls in Kasavanahalli - 7001305949 with real photos and phone numbers
Book Call Girls in Kasavanahalli - 7001305949 with real photos and phone numbersBook Call Girls in Kasavanahalli - 7001305949 with real photos and phone numbers
Book Call Girls in Kasavanahalli - 7001305949 with real photos and phone numbersnarwatsonia7
 
Russian Call Girls Gunjur Mugalur Road : 7001305949 High Profile Model Escort...
Russian Call Girls Gunjur Mugalur Road : 7001305949 High Profile Model Escort...Russian Call Girls Gunjur Mugalur Road : 7001305949 High Profile Model Escort...
Russian Call Girls Gunjur Mugalur Road : 7001305949 High Profile Model Escort...narwatsonia7
 
High Profile Call Girls Jaipur Vani 8445551418 Independent Escort Service Jaipur
High Profile Call Girls Jaipur Vani 8445551418 Independent Escort Service JaipurHigh Profile Call Girls Jaipur Vani 8445551418 Independent Escort Service Jaipur
High Profile Call Girls Jaipur Vani 8445551418 Independent Escort Service Jaipurparulsinha
 

Último (20)

VIP Call Girls Lucknow Nandini 7001305949 Independent Escort Service Lucknow
VIP Call Girls Lucknow Nandini 7001305949 Independent Escort Service LucknowVIP Call Girls Lucknow Nandini 7001305949 Independent Escort Service Lucknow
VIP Call Girls Lucknow Nandini 7001305949 Independent Escort Service Lucknow
 
call girls in munirka DELHI 🔝 >༒9540349809 🔝 genuine Escort Service 🔝✔️✔️
call girls in munirka  DELHI 🔝 >༒9540349809 🔝 genuine Escort Service 🔝✔️✔️call girls in munirka  DELHI 🔝 >༒9540349809 🔝 genuine Escort Service 🔝✔️✔️
call girls in munirka DELHI 🔝 >༒9540349809 🔝 genuine Escort Service 🔝✔️✔️
 
Call Girls Jp Nagar Just Call 7001305949 Top Class Call Girl Service Available
Call Girls Jp Nagar Just Call 7001305949 Top Class Call Girl Service AvailableCall Girls Jp Nagar Just Call 7001305949 Top Class Call Girl Service Available
Call Girls Jp Nagar Just Call 7001305949 Top Class Call Girl Service Available
 
Hematology and Immunology - Leukocytes Functions
Hematology and Immunology - Leukocytes FunctionsHematology and Immunology - Leukocytes Functions
Hematology and Immunology - Leukocytes Functions
 
Call Girls Hebbal Just Call 7001305949 Top Class Call Girl Service Available
Call Girls Hebbal Just Call 7001305949 Top Class Call Girl Service AvailableCall Girls Hebbal Just Call 7001305949 Top Class Call Girl Service Available
Call Girls Hebbal Just Call 7001305949 Top Class Call Girl Service Available
 
Call Girls Hsr Layout Just Call 7001305949 Top Class Call Girl Service Available
Call Girls Hsr Layout Just Call 7001305949 Top Class Call Girl Service AvailableCall Girls Hsr Layout Just Call 7001305949 Top Class Call Girl Service Available
Call Girls Hsr Layout Just Call 7001305949 Top Class Call Girl Service Available
 
Russian Call Girl Brookfield - 7001305949 Escorts Service 50% Off with Cash O...
Russian Call Girl Brookfield - 7001305949 Escorts Service 50% Off with Cash O...Russian Call Girl Brookfield - 7001305949 Escorts Service 50% Off with Cash O...
Russian Call Girl Brookfield - 7001305949 Escorts Service 50% Off with Cash O...
 
Kolkata Call Girls Services 9907093804 @24x7 High Class Babes Here Call Now
Kolkata Call Girls Services 9907093804 @24x7 High Class Babes Here Call NowKolkata Call Girls Services 9907093804 @24x7 High Class Babes Here Call Now
Kolkata Call Girls Services 9907093804 @24x7 High Class Babes Here Call Now
 
Call Girls Service Nandiambakkam | 7001305949 At Low Cost Cash Payment Booking
Call Girls Service Nandiambakkam | 7001305949 At Low Cost Cash Payment BookingCall Girls Service Nandiambakkam | 7001305949 At Low Cost Cash Payment Booking
Call Girls Service Nandiambakkam | 7001305949 At Low Cost Cash Payment Booking
 
Call Girl Nagpur Sia 7001305949 Independent Escort Service Nagpur
Call Girl Nagpur Sia 7001305949 Independent Escort Service NagpurCall Girl Nagpur Sia 7001305949 Independent Escort Service Nagpur
Call Girl Nagpur Sia 7001305949 Independent Escort Service Nagpur
 
Housewife Call Girls Bangalore - Call 7001305949 Rs-3500 with A/C Room Cash o...
Housewife Call Girls Bangalore - Call 7001305949 Rs-3500 with A/C Room Cash o...Housewife Call Girls Bangalore - Call 7001305949 Rs-3500 with A/C Room Cash o...
Housewife Call Girls Bangalore - Call 7001305949 Rs-3500 with A/C Room Cash o...
 
Call Girls Frazer Town Just Call 7001305949 Top Class Call Girl Service Avail...
Call Girls Frazer Town Just Call 7001305949 Top Class Call Girl Service Avail...Call Girls Frazer Town Just Call 7001305949 Top Class Call Girl Service Avail...
Call Girls Frazer Town Just Call 7001305949 Top Class Call Girl Service Avail...
 
See the 2,456 pharmacies on the National E-Pharmacy Platform
See the 2,456 pharmacies on the National E-Pharmacy PlatformSee the 2,456 pharmacies on the National E-Pharmacy Platform
See the 2,456 pharmacies on the National E-Pharmacy Platform
 
Russian Call Girls Chickpet - 7001305949 Booking and charges genuine rate for...
Russian Call Girls Chickpet - 7001305949 Booking and charges genuine rate for...Russian Call Girls Chickpet - 7001305949 Booking and charges genuine rate for...
Russian Call Girls Chickpet - 7001305949 Booking and charges genuine rate for...
 
Call Girls In Andheri East Call 9920874524 Book Hot And Sexy Girls
Call Girls In Andheri East Call 9920874524 Book Hot And Sexy GirlsCall Girls In Andheri East Call 9920874524 Book Hot And Sexy Girls
Call Girls In Andheri East Call 9920874524 Book Hot And Sexy Girls
 
Glomerular Filtration and determinants of glomerular filtration .pptx
Glomerular Filtration and  determinants of glomerular filtration .pptxGlomerular Filtration and  determinants of glomerular filtration .pptx
Glomerular Filtration and determinants of glomerular filtration .pptx
 
Call Girls Kanakapura Road Just Call 7001305949 Top Class Call Girl Service A...
Call Girls Kanakapura Road Just Call 7001305949 Top Class Call Girl Service A...Call Girls Kanakapura Road Just Call 7001305949 Top Class Call Girl Service A...
Call Girls Kanakapura Road Just Call 7001305949 Top Class Call Girl Service A...
 
Book Call Girls in Kasavanahalli - 7001305949 with real photos and phone numbers
Book Call Girls in Kasavanahalli - 7001305949 with real photos and phone numbersBook Call Girls in Kasavanahalli - 7001305949 with real photos and phone numbers
Book Call Girls in Kasavanahalli - 7001305949 with real photos and phone numbers
 
Russian Call Girls Gunjur Mugalur Road : 7001305949 High Profile Model Escort...
Russian Call Girls Gunjur Mugalur Road : 7001305949 High Profile Model Escort...Russian Call Girls Gunjur Mugalur Road : 7001305949 High Profile Model Escort...
Russian Call Girls Gunjur Mugalur Road : 7001305949 High Profile Model Escort...
 
High Profile Call Girls Jaipur Vani 8445551418 Independent Escort Service Jaipur
High Profile Call Girls Jaipur Vani 8445551418 Independent Escort Service JaipurHigh Profile Call Girls Jaipur Vani 8445551418 Independent Escort Service Jaipur
High Profile Call Girls Jaipur Vani 8445551418 Independent Escort Service Jaipur
 

Derm book

  • 1. CLINICAL CASES AND PEARLS BENJAMIN BARANKIN, MD, FRCPC ANIL KURIAN, MD
  • 2. RAPID FIRE DERMATOLOGY Dr. Benjamin Barankin is a Toronto dermatologist, the Founder & Medical Director of Toronto Dermatology Centre (www.torontodermatologycentre.com). He is also the author of other dermatology books, including Derm Notes, Stedman’s Illustrated Dictionary of Dermatology Eponyms, Dermographies Volume 1 & 2, and Diagnostic Criteria in Dermatology. DR. ANIL KURIAN is a dermatology resident at the University of Alberta in Edmonton. At this early stage in his career, he has already co-authored more than 25 dermatology-related articles and also received a prestigious award from the Ontario Medical Association (OMA) for his patient advocacy work promoting a ban on indoor tanning services to minors.
  • 3. Praise for Rapid Fire Dermatology “Rapid Fire Dermatology by Drs. Barankin and Kurian is an excellent state-of-the art guide to many of the common skin disorders that primary providers will encounter with regularity. The format differs from most other guides in that this book is actually fun to read. It also provides realistic medication and treatment options that are applicable to the primary care setting. It will have broad appeal to its targeted audience.” — Dr. David J. Elpern, Dermatologist, Williamstown, MA USA “Rapid Fire Dermatology is a wonderful learning tool and reference book for family physicians. Focusing on the most common dermatological conditions seen in a family physician office setting, this book offers clear pictures and an intriguing case presentation approach that allows you to test your knowledge with multiple choice questions. The explanations are clearly explained with a wonderful practical approach to treating these dermatological conditions. The treatment options are gauged at the family practice level, and more advanced dermatological approaches for more complex cases are discussed as well. It was an enjoyable learning experience to read this book.” — Dr. Monica Pearl, Family Physician, Toronto, ON Canada “Clinical cases are a key component in modern medical education, assisting the clinician to work through cases using best practice techniques. Dermatology is a highly visual subject requiring the health care provider to understand often very subtle differences in the presentation of patients. This new book will help define accurately the diagnostic and management criteria to base clinical decision-making on. I am very excited about the potential for this clinical reference tool!” — Dr. Robert Norman, Dermatologist, Associate Professor, Department of Dermatology, University of Central Florida College of Medicine, Tampa Bay, FL USA “Barankin and Kurian’s new book, Rapid Fire Dermatology, is an outstanding work for those interested in learning the basics of dermatology. The book focuses on some of the most common afflictions of men and women, providing practical advice on both differential diagnosis and management. The clinical photographs are wonderfully illustrative of common disease presentations, and the text is concise and easily read. If you are a family or other primary care provider, a resident or a medical student, you will see patients with these conditions. Rapid Fire Dermatology will prepare you to recognize and manage them.” — Dr. Steven R. Feldman, Dermatologist, Professor of Dermatology, Pathology & Public Health Sciences, Wake Forest University School of Medicine, Winston-Salem, NC USA ii
  • 4. “The case-based Rapid Fire Dermatology is a great way to learn a wide variety of the most common skin diseases. The pictures are terrific and the text is concise and to the point. I highly recommend this book.” — Dr. Gary Goldenberg, Dermatologist, Assistant Professor, Dermatology and Pathology, Medical Director of the Dermatology Faculty Practice, Mount Sinai School of Medicine, New York, NY USA “Drs. Barankin and Kurian have put together the perfect pocket summary of the most common dermatologic conditions that a family physician, pediatrician, or nurse practitioner will see on a regular basis. Not only does the small book literally fit easily within a typical white coat pocket, it details 25 of the most common skin conditions seen in general practice using case presentations and Q&A format that allows for the rapid assimilation of the crucial information needed to make a definitive diagnosis, initiate appropriate therapy, or recognize when a referral to a dermatologist is warranted. This book is the ideal entry level text to help demystify the confusing world of dermatology to the primary care physician and other healthcare providers.” — Dr. Thomas G. Salopek, Dermatologist, Director, Associate Professor, Division of Dermatology & Cutaneous Sciences at University of Alberta, Edmonton, AB Canada Copyright © 2012 Benjamin Barankin and Anil Kurian All rights reserved. Except for the quotation of short passages for review purposes, no part of this publication may be reproduced in any form without prior permission of the authors. Print Design: Brenda Conroy
 iBook Design: Paul La Rue [Fission Media] Printed Copies of this book may be obtained from:
 Dr. Benjamin Barankin at benbarankin@gmail.com Library and Archives Canada Cataloguing in Publication iii
  • 5. Authors Dr. Benjamin Barankin is a Toronto dermatologist specializing in medical, surgical, laser and cosmetic care of the skin. His undergraduate university training was in Life Sciences and Psychology at Queen’s University, followed by medical school at the University of Western Ontario, and dermatology residency at the University of Alberta. Dr. Benjamin Barankin is also a widely published author with more than 500 articles related to dermatology and he has gained international distinction as the author of 6 other books on dermatology. He has been honoured with being voted onto the Executive Board of the Canadian Dermatology Association (CDA) where he held the position of Secretary and Chair of the Membership & Awards Committee. He serves on the editorial boards of numerous medical journals and newspapers, and Dr. Barankin is also team dermatologist for the Toronto FC soccer team. He resides in Toronto with his physician wife and daughters. Dr. Anil Kurian is a dermatology resident at the University of Alberta. Prior to entering medicine, he had a career in nursing, working as a registered nurse and subsequently, a nurse practitioner. He completed his Bachelor of Science in Nursing degree from the University of Ottawa and his Master of Nursing degree from Dalhousie University. He completed his medical school training at McMaster University. Dr. Anil Kurian is already making an impact in the field of dermatology. At this early stage in his medical career, he has already co-authored more than 25 dermatology-related articles and also received a prestigious award from the Ontario Medical Association (OMA) for his patient advocacy work promoting a ban on indoor tanning services to minors. He is also project lead of many groundbreaking dermatology projects in Edmonton, including the innovative use of teledermatology in the ER department, the first of its kind in Canada. He resides in Edmonton with his family. iv
  • 6. Introduction The idea of this book was developed through the very successful case-based columns and articles written by the book’s senior author, Dr. Benjamin Barankin, over the past decade. Dr. Barankin has spoken at many primary care conferences and aspires to educate family physicians about how important recognizing and treating dermatological conditions are in a primary care setting. Incorporating routine skin checks during a patient’s physical examination promotes greater surveillance for the diagnosis of pathological skin conditions and lesions, as well as proper treatment being implemented in a timely fashion. We aspired to integrate a large number of dermatological casebased conditions packaged into one succinct pocket-style book with quality dermatology images. Each case will be described in great detail and will provide the clinical practitioner with specific tips on the types of dermatological visual and tactile cues they should recognize and look for in each particular case, to improve their diagnostic acumen. We envisioned a practical book formatted to include exclusive Canadian medical content. We believe that this book would be relevant to family physicians and pediatricians, for use as a pocket reference in their clinic-based practices, as well as medical residents and students wishing to become familiar with various dermatological presentations. Case-based scenarios are a well-received and utilized style of learning among physicians and trainees. Dermatology is a highly visual field within medicine and recognition of visual cues and patterns, in conjunction with the clinical patient history, direct the practitioner to the correct diagnosis. The 25 dermatology cases presented within are common and important clinical concerns of patients that include skin cancers, skin infections, inflammatory and immunologic skin conditions, as well as benign lesions. Additionally, each dermatology case will have an interactive question and answer (Q&A) section where the reader can test their knowledge relating to the clinical case presented. This unique and interactive self-test will aid the reader to retain the primary teaching points of each of the cases featured. Rapid Fire Dermatology is a unique dermatology book tailored to the Canadian marketplace, including up-to-date, comprehensive information regarding dermatological diagnosis, as well as specific medication and treatment recommendations. Additionally, after each case presentation, there is an added “Clinical Pearls” section with informative dermatologist tips related to diagnosis or treatment aspects for that particular condition. Rapid Fire Dermatology is a great resource for Canadian physicians, medical residents or students. We hope you find it of value to your practice. If you have any comments about this book or suggestions for topics for the next edition, please contact Dr. Benjamin Barankin at: benbarankin@gmail.com Acknowledgements The authors would like to thank their respective families for their unconditional support of educational pursuits by the authors, specifically relating to dermatology education and research endeavours. We are also grateful for the efforts of our book designer, Brenda Conroy. The development and funding of this book has been made possible through a grant provided by Leo Pharma Inc. v
  • 7. Question 1 of 3 1. What is her diagnosis? A. Allergic contact dermatitis B. Psoriasis C. Tinea corporis T HIGH RASH WITH CRUSTY LESIONS A 15 year-old girl presents with a mild fever and a moderately pruritic rash on her left and right thigh regions. The lesions began 3 days before as blisters discharging a clear yellowish fluid. The blisters progressively enlarged despite treatment with a topical antibiotic ointment. She is taking no other medications and her medical history is unremarkable. D. Bullous impetigo E. Bullous pemphigoid Check Answer Diagnosis & Information 6
  • 8. Predisposing factors include skin abrasions, minor trauma, burns, poor hygiene, insect bites, diabetes mellitus, primary varicella infection, and pre-existing skin disease, such as eczema. Also, those with nasal or perineal Staphylococcal aureus colonization are at increased risk for developing impetigo. Diagnosis: Bullous impetigo Bullous impetigo commonly starts with small vesicles on the face, trunk, buttocks, perineum, or extremities, without surrounding erythema. These vesicles rapidly enlarge into flaccid, transparent bullae measuring from 1 to 5 cm in diameter. Bullae initially contain a clear yellow fluid that subsequently becomes dark yellow and turbid. Their margins are sharply demarcated, having no erythematous halo. The lesions heal more quickly than nonbullous lesions. The condition might be associated with fever, diarrhea, and weakness, but unlike nonbullous impetigo it is not associated with lymphadenopathy. Impetigo is a superficial skin infection encountered most frequently among children. It typically presents with multiple vesicular lesions on an erythematous base, which eventually crust over. There are 2 types: nonbullous and bullous. Both forms involve only the most superficial layers of the skin. The Nikolsky sign (sloughing of the epidermis with sliding pressure) is usually negative. Complications of impetigo include lymphadenitis, cellulitis, sepsis, or glomerulonephritis. The nonbullous form is the most common and is likely to be caused by a mixed staphylococcal and streptococcal infection. The bullous form, however, is a highly contagious condition caused by an epidermolytic toxin (produced most commonly by staphylococci) at the site of infection. Cultures of fluid from an intact blister or moist plaque should yield the causative agent; if the patient appears ill, blood cultures should also be obtained. The differential diagnosis primarily includes all of the herpes simplex viral infections, allergic contact dermatitis, thermal burns, bullous insect bite reactions, bullous pemphigoid and rarely pemphigus. 7
  • 9. Prophylactic treatments to prevent staph infections include ‘bleach baths’. This is done by adding ¼ to ½ cup of common liquid bleach to approximately 40 gallons of bath water. Soaking in the bleach bath for about 10 minutes (2-3 times per week over 1-2 weeks), is an effective way to prevent the occurrence of cutaneous staphylococcal infections. This is especially helpful in children with recurrent impetigo or regular eczema flares. Local treatment of cutaneous lesions can be achieved with 2% mupirocin ointment (Bactroban®) or fusidic acid 2% cream (Fucidin® cream) and sodium fusidate 2% ointment (Fucidin® ointment). The former should be prescribed tid for 8-12 days, while the latter can be used 2-4 times daily for 8-12 days. In addition, local care, which includes cleansing, removal of crusts, and application of wet dressings, is sufficient to cure mild cases. Some studies have demonstrated that 2% mupirocin ointment is as safe and effective as oral erythromycin in the treatment of patients with impetigo, while another study showed that 2% fusidic acid was more effective than oral clindamycin, erythromycin or flucloxacillin in the treatment of soft tissue infections. It is important that duration of either individual topical therapy alone be limited as bacterial resistance can be seen with both local treatment options. Also, adding a mild topical steroid to antibiotic therapy, such as fusidic acid plus hydrocortisone found in Fucidin®H, is beneficial to deal with the underlying dermatitis and pruritus. Complicated, quickly spreading or widespread cases of bullous impetigo require 5 to 10 days of oral β-lactamase– resistant antibiotics, such as cephalexin (Keflex®) 250-500mg qid for 10 days or amoxicillin/clavulanate (Augmentin®) 250-500mg bid for 10 days. Clinical Pearl • Impetigo is highly contagious; patients should not touch the lesions or be discouraged by covering the lesion. The child should be kept at home until the lesions resolve if in an exposed area that other children may come in contact with. “When you are looking for the cause of disease, think food, bugs and drugs.” 
 ~ Drs. Walter & Dorinda Shelley, American Dermatologists and authors of Advanced Dermatologic Diagnosis 8
  • 10. Question 1 of 3 What is the most likely diagnosis? A. Tinea corporis B. Sarcoidosis C. Granuloma annulare R EDDISH - BROWN LESIONS ON DORSAL HAND A 36 year-old male presents with erythematous-brown asymptomatic plaques on his dorsal hands. He is bothered by the cosmetic appearance, and would like to stop having to cover up the areas because of embarrassment. He occasionally takes ibuprofen and acetaminophen for tension headaches. D. Lichen planus E. Erythema annulare centrifugum Check Answer Diagnosis & Information 9
  • 11. GA is an idiopathic condition, with controversial associations with infections, diabetes, and thyroid disease; there appears to be a hereditary component in some cases. Localized trauma and sunlight have also been hypothesized to play a role in its etiology. GA is often a clinical diagnosis. A punch or incisional biopsy is recommended for atypical or uncommon presentations, and especially if growing quickly and tender. Diagnosis: Granuloma annulare Granuloma annulare (GA) is a benign, inflammatory skin condition of dermal papules and annular plaques commonly affecting acral sites. GA occurs most commonly in women and in younger individuals (typically less than 30 years old). There are several clinical variants, including: localized GA (most common, especially in children & young adults), generalized GA (10%), subcutaneous GA, perforating GA (5%), and arcuate dermal erythema. Localized GA typically manifests as groups of small erythematous to brown papules, often in an annular arrangement (raised border, central clearing) over distal extremities, especially the hands, feet, and extensor limbs. Management involves reassurance that the condition is benign and spontaneous resolution of lesions typically occurs within 2 years in 50% of patients, especially of the localized clinical variant. Various treatment options have been utilized, but often have suboptimal clinical efficacy. Intralesional triamcinolone acetonide (Kenalog®, 2.5-5mg/ml q 4-6 weeks) is the most uniformly successful therapy. Potent topical steroids with or without occlusion can also be tried but have less efficacy. Cryotherapy in lighter skin types can also provide benefit alone or more commonly in combination with intralesional Kenalog®. While localized GA resolves after an average of 2 years duration, generalized forms of GA typically have a more chronic duration and unfortunately, are more resistant to treatment options. Phototherapy and oral retinoids are commonly employed in cases of generalized GA, and increasingly biologics are being used. Clinical Pearls • Granuloma annulare can be a frustrating condition to manage, as it is one of those conditions that develops for reasons that are not well understood, and similarly often resolves spontaneously. • Interestingly, some cases have been reported to disappear when oral antibiotics were given for sinus, fungal or other infections. 10
  • 12. “Just because you have a diagnostic handle on a disease doesn’t mean you can handle it.” 
 ~ Drs. Walter & Dorinda Shelley, American Dermatologists and authors of Advanced Dermatologic Diagnosis 11
  • 13. Question 1 of 3 What is the most likely diagnosis? A. Pilomatricoma B. Epidermoid cyst C. Milia cyst S LOW GROWING BACK LUMP A 64 year-old male presents to clinic with a slowly growing nodule on his back. It has been present for at least a couple years, is asymptomatic, and has never had any bleeding or discharge. He has never had such a lesion before. D. Dermoid cyst E. Lipoma Check Answer Diagnosis & Information 12
  • 14. pilosebaceous unit, HPV infection, or traumatic implantation of epidermal elements. A number of hereditary syndromes including nevoid basal cell carcinoma syndrome and Gardner syndrome have the feature of multiple epidermoid cysts. As well, idiopathic scrotal calcinosis may represent the end stage (calcification) of scrotal epidermoid cysts. Patients often present to clinic for a number of reasons relating to an epidermoid cyst. Reasons include the cosmetic appearance of a visible body ‘lump’, the malodorous cheese/curd-like material that can be extruded from the lesion, or because of lesional discomfort associated with either inflammation or infection. Clinically, the lesions appear as round, firm, flesh-coloured nodules (of various sizes), which quite often have a central punctum. Commonly affected areas on the body include the head, neck and trunk. Diagnosis: Epidermoid cyst Epidermoid cysts form as a result of the growth of epidermal cells within a focal area of the dermis. The material inside the cysts is chemotactic for neutrophils and can result in inflammation. Most epidermoid cysts do not cause any problems and grow rather slowly. However, cysts can become inflamed and less commonly infected which results in redness, tenderness and pain, due to trauma or other factors. Epidermoid cysts are more common in men during the young adulthood years but may be noted at any age. The pathogenesis of an epidermoid cyst is not completely clear, but some factors that may play a role include occlusion of the The diagnosis of an epidermoid cyst is typically made clinically. If infection of a cyst is of concern, a swab for bacterial culture can be sent. Diagnostic imaging is rarely necessary unless the lesion is in the midline of the body. In such a case, an ultrasound can be considered. Epidermoid cysts are benign lesions and require treatment only if symptomatic or for cosmetic purposes. A tender, inflamed, uninfected cyst often benefits from an intralesional triamcinolone acetonide (Kenalog®) injection. For cysts presumed or proven to be infected, incision and drainage is recommended following by a course of oral gram positive coverage antibiotics (eg. Keflex® 500mg qid x 7-10 days). Epidermoid cysts are excised using a classic large incision, mini incision, or trephination with a punch biopsy device. Incising and draining is a quick procedure, but has high recurrence risk since 13
  • 15. the entire cyst wall is unlikely to be evacuated. Excision of the entire cyst, including the epidermal cyst wall lining, is the most definitive treatment, but even still recurrences may occur. Clinical Pearl • When removing epidermoid cysts, a punch biopsy instrument can be used that can result in a smaller scar than doing an elliptical excision. Along with trying to extrude the contents with forceps and pressure and undermining, inserting and moving back and forth a curette can be helpful in removing any remaining cyst wall contents to prevent recurrence. “Sometimes you have to wait for a disease to grow up before a diagnosis can be made.”
 ~ Drs. Walter & Dorinda Shelley, American Dermatologists and authors of Advanced Dermatologic Diagnosis 14
  • 16. Question 1 of 3 What is the most likely diagnosis? A. Pityriasis alba B. Tinea versicolor C. Post-inflammatory hypopigmentation W HITE PATCH OVER KNEE A 40 year-old female presents with a two-year history of a white patch on her anterior knee and lower leg area. She has smaller white macules of this on her fingers and ankle. These lesions are asymptomatic. She has a history of pernicious anemia and a family history of thyroid disease. D. Vitiligo E. Nevus anemicus Check Answer Diagnosis & Information 15
  • 17. Screening for these co-morbidities with periodic blood tests should be considered when a diagnosis of vitiligo is made. Vitiligo progresses without symptoms, though early lesions may occasionally be pruritic. Vitiligo presents as sharply demarcated patches that are cosmetically disturbing to most patients, especially those with darker skin colour where there is more contrast. After a few small white lesions appear, they increase in number and size, becoming confluent and having unusual shapes. Vitiligo is classified as having localized or generalized involvement. Although any area of skin can be affected, the most commonly affected areas include: face, neck, bony prominences, dorsal hands, and fingers. Hair in a patch of vitiligo will also depigment. A Wood’s lamp examination can help clarify the diagnosis as the depigmented white areas of vitiligo become accentuated under the light. Diagnosis: Vitiligo Vitiligo is an acquired, benign, depigmentation of the skin caused by destruction of melanocytes in skin and hair, resulting in white macules and patches. It affects 1-2% of the population with an equal gender distribution and most commonly onsets between 10-30 years of age. The etiology of vitiligo has not been fully elucidated, but the pathogenesis is multifactorial, likely having both genetic and autoimmune mechanisms. Melanocytes are destroyed in the skin and those in hair follicles can also be affected resulting in white hair. Vitiligo has been associated with other autoimmune conditions such as: pernicious anemia, alopecia areata, diabetes mellitus, Addison’s disease, and thyroid disease. Management options include potent topical steroids and topical calcineurin inhibitors (especially on areas with thin skin, such as the face or intertriginous areas). Phototherapy is commonly employed for more widespread involvement or resistant cases. Less commonly, excimer laser, repigmentation surgery, and depigmentation therapies are employed. A discussion of the use of cover up makeup (e.g. CoverFX, Dermablend™, Covermark®) and topical tanning creams should be considered. Management can be frustrating as improvement is slow and unpredictable. 16
  • 18. Clinical Pearls • Vitiligo is a chronic and difficult condition to manage. Fortunately, areas such as the face typically respond well to treatment. Hands and feet are notoriously slow to respond to therapy. • Some dermatologists believe there is benefit to taking folic acid, vitamin C, and vitamin B12 in the treatment of vitiligo. Some have also observed improvement with long-term use of ibuprofen or acetylsalicylic acid. “The darker the skin, the more prominent the paler patches. 
 But for people of every shade, vitiligo can cause distress as strangers gawk and job interviews become sidetracked.”
 ~ Catherine Saint Louis, Journalist, New York Times 17
  • 19. Question 1 of 3 What is the most likely diagnosis? A. Compound nevis B. Angiofibroma C. Basal cell carcinoma C HRONIC ERYTHEMATOUS FACIAL PAPULE A 74 year-old female presents with an erythematous and telangiectactic papule to her nose. She has been using topical steroid and antifungal creams for the past 2 months with minimal clinical improvement. D. Dermatofibroma E. Squamous cell carcinoma Check Answer Diagnosis & Information 18
  • 20. previous actinic keratosis. Chronic sun exposure in a fair skin type is the major risk factor for the development of BCC. The most common sites are the head (especially nose) and neck, trunk, and arms and legs. Clinical presentation can vary based on subtype, which include: nodular BCC, superficial BCC, pigmented BCC, cystic BCC, and morpheaform/sclerosing BCC. BCC is largely a clinical diagnosis in many cases and no additional work-up is required. Diagnostic accuracy is enhanced by good lighting and magnification with a dermatoscope may be helpful. A biopsy of the lesion is indicated when clinical doubt as to the diagnosis exists or when the histological subtype of BCC may influence treatment selection and prognosis. Diagnosis: Basal cell carcinoma (BCC) This is the most common malignancy in humans, typically appearing on sun-exposed skin. BCCs can result in significant local destruction and even disfigurement, especially if located around the nose or eyes. They rarely metastasize (<0.1%). In Caucasians, there is approximately 25-35% lifetime risk of developing a BCC. These lesions are uncommon in dark skin types. Likelihood of BCCs increases with age, and they are considered uncommon under 35 years of age. Risk factors for BCC include a fair complexion, red or blond hair, light eye color, increased sun exposure during childhood years, ionizing radiation, UVA radiation, immunosuppression and BCC is a highly treatable form of skin cancer. Treatment of BCC can be surgical or nonsurgical. Surgical approaches with most definitive 5-year cure rates include surgical excision and Mohs micrographic surgery (99%) followed by curettage and electrodessication and cryosurgery (95%). Nonsurgical approaches include topical imiquimod, radiotherapy, and photodynamic therapy. Preventive measures for BCC include avoidance of the sun during peak hours and protection against sun exposure with a hat, clothing, and broad spectrum sunscreens. Avoiding tanning salons is also a must. Clinical Pearl • A patient with a history skin cancer complaining of a “sore that doesn’t heal” or “the lesion has scabbed and/or bled on a few occasions but won’t go away” is highly suggestive of the diagnosis of BCC. 19
  • 21. “Inject xylocaine or saline into a suspect basal cell carcinoma to enhance its gross morphology.” 
 ~ Drs. Walter & Dorinda Shelley, American Dermatologists and authors of Advanced Dermatologic Diagnosis 20
  • 22. Question 1 of 3 . What is your diagnosis? A. Melasma B. Lentigines C. Post-inflammatory hypermelanotic macules D ARK PATCHES ON CHEEKS A 42 year-old woman presented with brown macules that she has had for several years on both cheeks. She began to notice the macules during her pregnancy of her first child 4 years ago. She currently takes no medications, and her medical history is otherwise unremarkable. D. Addison’s disease E. Discoid lupus erythematosus Check Answer Diagnosis & Information 21
  • 23. with darker complexions, including those of Hispanic, Middle Eastern, Asian, and Indian ethnicities. While most cases are idiopathic, melasma has also been associated with genetics, exposure to ultraviolet radiation, pregnancy, oral contraceptives, hormone replacement therapies, mild endocrine disturbances (e.g. ovarian or thyroid dysfunction), cosmetics, antiepileptic medications, and other photosensitizing drugs. Sun exposure tends to be the most important causative factor in all patients, so management options for melasma must always include avoidance of excessive exposure to sunlight. Diagnosis: Melasma Melasma is a common hypermelanosis that typically occurs on sun-exposed areas of the face. Melasma is characterized by symmetrically distributed macules with irregular borders ranging from light brown to dark brown to gray-brown. Common clinically distinct distributions include centrofacial (forehead, nose, chin, upper lip), malar (cheeks), and mandibular (lower jaw). Forearms are also occasionally affected. Pigmentation typically slowly evolves over many weeks or months. Melasma is rare before puberty and is noted more commonly in women of reproductive age. Men comprise approximately 10% of the affected population. Melasma is more prevalent among people Wood’s lamp examination is useful in diagnosis and can help classify melasma into epidermal, dermal, and mixed types. Epidermal pigmentation is the most common type of melasma where pigmentation is accentuated under Wood’s light examination. Dermal pigmentation decreases under Wood’s light examination, and mixed types show areas with increased (epidermal), absent, or decreased (dermal) pigmentation. Classifying the melasma type is a valuable indicator of prognosis. While epidermal pigmentation tends to respond better and faster to hypopigmenting agents (eg. hydroquinone), dermal pigmentation lesions show variable response, often with much less patient satisfaction. Melasma is a cosmetic problem that is difficult to treat. Current treatments include hypopigmenting compounds, exfoliative agents, and lasers. Topical hypopigmenting agents, such as hydroquinone 2% to 10%, arbutinin, lumixyl, retinoic acid or tretinoin, azelaic acid, vitamin C, mild-to-moderate strength topical corticosteroids, and combinations of these agents are effective. Exfoliative agents, such as alpha-hydroxy acid chemical peels or microdermabrasion, are useful adjuncts to therapy. In 22
  • 24. recent years, various laser therapies have been used with mixed results. In some cases, melasma has spontaneously resolved several months postpartum or after cessation of oral contraceptives. If indicated, discontinuation of causative medications is appropriate. Treatment and prevention of melasma should always include avoiding excessive sun exposure and daily use of broad-spectrum sunscreens. Clinical Pearls • Granuloma annulare can be a frustrating condition to manage, as it is one of those conditions that develops for reasons that are not well understood, and similarly often resolves spontaneously. • Interestingly, some cases have been reported to disappear when oral antibiotics were given for sinus, fungal or other infections. “If you haven’t looked at it with the makeup off, 
 you haven’t looked at it.” 
 ~ Drs. Walter & Dorinda Shelley, American Dermatologists and authors of Advanced Dermatologic 23
  • 25. Question 1 of 3 What is your diagnosis? A. Psoriasis B. Nummular eczema C. Allergic contact dermatitis I TCHY BUMPS TO LOWER ABDOMEN A 36 year-old male presents with a pruritic papular eruption to his lower abdomen. He had mild acne as a teenager, but no other skin problems. He has not used any topical agents to treat his abdomen and takes a proton-pump inhibitor (PPI) to treat his gastroesophageal reflux. D. Tinea corporis E. Fixed drug eruption Check Answer Diagnosis & Information 24
  • 26. contact dermatitis”) as opposed to a specific sensitivity, in the case of ACD. Common causes of ACD include nickel, fragrances, preservatives, black hair dye, topical medications (e.g. neomycin, bacitracin), latex, rhus, cobalt, chromate, epoxy resin, rubber accelerators, rosin and many more chemical agents. Patients often complain of significant pruritus, and may have mild burning or discomfort. Lesions are typically edematous, erythematous, and welldemarcated. Diagnosis: Allergic contact dermatitis Allergic contact dermatitis (ACD) is a delayed type of induced sensitivity that occurs in individuals that have been previously sensitized to any of more than 3,000 chemicals, and is more likely in genetically predisposed individuals. The allergic reaction causes inflammation of the skin that is manifested by varying degrees of erythema, edema, and vesiculation. A reaction typically develops within 24-48hrs of a subsequent exposure, although it can occur up to 7 days later; it typically resolves after 2-3 weeks. The term “contact dermatitis” sometimes is used incorrectly as a synonym for ACD. Contact dermatitis is inflammation of the skin induced by chemicals that directly damage the skin (also known as “irritant Definitive treatment is the identification and removal of any potential causal agents; otherwise, the patient is at increased risk for chronic or recurrent dermatitis. Referral for skin patch testing (not skin prick testing; typically performed by a dermatologist) is appropriate. Online resources allow the physician to create a list of products free of allergens to which the patient is allergic, and should be avoided. Potent topical steroids are used, and occasionally if the reaction is severe or more widespread, a short course of oral steroids (e.g. Prednisone) can hasten resolution. Cool saline compresses, sedating antihistamines to aid in sleep, and topical immunomodulators (e.g. Protopic®, Elidel®) and emollients can provide benefit to the patient as well. 25
  • 27. Clinical Pearls • A pruritic rash localized to the infra-umbilical area is an allergic contact dermatitis to nickel until proven otherwise. If this rash is not treated, it can eventually spread out to other areas of the body in an immune phenomenon termed the “id reaction.” • If a jean snap is a trigger for nickel contact (common), apply 1 coat of red nail polish on the inside of the snap and also encourage tucking in of one’s shirt. Once the red nail polish has disappeared (e.g. typically after 2 washings/dryings of the jeans), the absence of the red colour will remind you to repaint the jean snap. You can also cover it with duct tape. “In a particular clinic, the incidence of allergic contact dermatitis is determined by the interest the dermatologist takes in allergic contact dermatitis.” 
 ~ Drs. N. Hjorth & S. Fregert, Swedish and Danish Dermatologists 26
  • 28. Question 1 of 3 What is the most likely diagnosis? A. Epidermoid cyst B. Milia cyst C. Digital mucous cyst U NCOMFORTABLE FINGER LESION A 50 year-old male is bothered by the appearance of a papule on his finger. It has grown very slowly over many months and doesn’t bleed. The lesion is tender only when it is firmly palpated or when he “accidentally bumps it.” D. Verruca vulgaris E. Cystic basal cell cancer Check Answer Diagnosis & Information 27
  • 29. yellow-tinged. The lesions, which are usually solitary and appear lateral to the midline, are more common on dominant hands. Toes are less commonly involved. Although usually asymptomatic, pain can occur if there is impingement on nerve fibers, and rarely larger cysts can disfigure the affected digit. Nail deformities occur in many cases – called the “nail groove” sign. Women are more commonly affected (more than twice as often as men), as are those individuals between the ages of 50 and 70. Diagnosis: Digital mucous cyst Digital mucous cysts (DMCs) are benign and often asymptomatic ganglion cysts of the digits, typically located at the distal interphalangeal (DIP) joints or at the proximal nail fold. The etiology is uncertain, although it may relate to mucoid degeneration of connective tissue; this process seems to involve communication with the adjacent DIP joint and often is associated with osteoarthritis, and less commonly with antecedent trauma. Digital mucous cysts are translucent to flesh-coloured, solitary, round and dome-shaped semi-firm papulonodules. The cysts can be from a few millimeters to approximately one centimeter in diameter. The cysts contain a gelatinous thick fluid that is clear or Surgery or pseudo-surgical options are the standard of therapy and attempt to remove the cyst and any underlying stalk connected to the joint. Using conservative therapies, one can periodically needle the cyst with a wide-bore needles resulting in drainage and scarification until the cyst resolves. Aspirating the cyst contents followed by intralesional triamcinolone acetonide (10mg/ml) or liquid nitrogen cryotherapy is commonly employed. Less commonly, curettage +/- electrodessication, CO2 laser, or sclerotherapy techniques can be utilized. More aggressive surgical procedures using cold-steel surgical excision have been used which may require flaps or grafts. Another approach involves marsupialization, or excision of the whole proximal nail fold, with subsequent healing by secondary intention. Aggressive surgery to remove underlying osteophytes at the joint results in few recurrences. However, this procedure may slightly increase the risk of surgical complications (e.g. joint stiffness, reduced movement). Osteophyte removal alone (without removal of the cyst) has been reported to be effective. Nail deformities often resolve following osteophyte removal. While osteophyte 28
  • 30. removal or open joint procedures are usually performed in the hospital setting, other management techniques can be performed in an office setting by primary care providers and dermatologists. Clinical Pearl • Puncturing a few holes into the lesion with a 25guage needle, extruding most of the viscous contents, and then applying cryotherapy is effective in many cases. As a backup, the patient can be given the needle that was used and instructed that if there is a recurrence, to boil the needle and repuncture the cyst and squeeze out its contents, eventually leading to permanent scarification and an end to the cyst. “Do not show your wounded finger, for everything will knock up against it.” ~ Baltasar Garcian, Spanish philosopher 29
  • 31. Question 1 of 3 What is your diagnosis? A. Pigmented basal cell carcinoma B. Melanoma C. Dysplastic nevus D. Congenital melanocytic nevus D ARK SPOT ON CHEEK A 41 year-old woman presents with a dark lesion to her right cheek. The lesion causes no discomfort and it does not bleed or itch. The patient is not exactly sure how long the lesion has been present, but feels it has been at least a few months. She has a personal history of mild facial acne as a teenager and has no family history of skin problems. The patient worked as a lifeguard when she was a teenager and recalls several painful sunburns. She does not take any medications and has no known drug allergies. E. Blue nevus Check Answer Diagnosis & Information 30
  • 32. respectively. However, mortality rates are higher in darker skin types due often to a more advanced presentation. There is little gender discrepancy. Older persons are more likely to acquire and die from melanoma, with a median age at diagnosis of 53. Primary risk factors for or clinical warning signs of melanoma include the following: • Changing mole (most important clinical warning sign; 
 the “E in Evolution”) • Clinical atypical/dysplastic nevi (particularly >5-10) • Large numbers of common nevi (>100) • Large (giant) congenital nevi (>20 cm diameter in an adult) • Previous melanoma • Sun sensitivity/history of excessive sun exposure Diagnosis: Melanoma • Melanoma in first-degree relative(s) Melanoma is a skin cancer of pigment-producing melanocytes that typically affects the skin, and less commonly the mucosae, eyes, leptomeninges and gastrointestinal tract. It account for approximately 5% of skin cancers, but 75% of deaths from skin cancer. It is believed that two-thirds of melanomas arise de novo, while one-third develop from pre-existing nevi. In Canada, approximately 5,000 people are diagnosed with melanoma each year, and approximately 950 will die of the disease. Detection and treatment in early stages provides the best opportunity for cure. • Prior nonmelanoma skin cancer (basal cell and squamous 
 cell carcinoma) • Male sex • Age older than 50 years • Presence of xeroderma pigmentosum or familial atypical 
 mole melanoma syndrome: These 2 genodermatoses confer a
 500- to 1000-fold greater relative risk of developing
 melanoma. Caucasians are most commonly afflicted, with black and Hispanic persons having one twentieth and one-sixth the prevalence, 31
  • 33. The most common warning signs are a new or changing mole. Variation in the ABCDs (Asymmetry, Border irregularity, Color variegation, Diameter >6mm) is evident in more than 80% of lesions at diagnosis. Less commonly, one may note symptoms such as pruritus, pain, bleeding or ulceration. The letter “E” has been added to the ABCDs to signify “evolution” or change in an existing lesion. As well, the “ugly duckling” warning sign which signifies concern for a lesion that looks different from the rest is a beneficial adjunct to the ABCDE rule. The four main clinico-histopathologic subtypes of primary cutaneous melanoma are: superficial spreading melanoma (most common on trunk in men, and legs of women), nodular melanoma (15-30% of cases), lentigo maligna melanoma (slow-growing, usually head & neck of elderly), acral lentiginous melanoma (occurs on palms, soles or under nail plate; more common in darker skin types). Clinical Pearls • Any new black lesion on the skin should be strongly considered for biopsy or referral to a dermatologist. • Even after melanoma wide excision, these patients and those persons in their genetic lineage should be screened periodically for life and a much higher index of suspicion for biopsy is warranted for any new lesion or dark lesion on their skin. “It is easy to know a nevus. But to know a melanoma takes suspicion and excision, and remember that close observation can help prevent a lentiginous lesion from becoming a ‘litigenous’ one.” 
 ~ Drs. Walter & Dorinda Shelley, American Dermatologists and authors of Advanced Dermatologic Diagnosis If detected early, melanoma can be cured with surgical excision. Stage 1 lesions (melanoma in situ) confer a 5 year survival rate of 95%, whereas, stage IV melanoma metastatic to skin, subcutaneous tissue or lymph nodes with normal LDH has a 5year survival rate of 19%. A stage IV melanoma that has metastasized to lungs has a 5 year survival rate of 7%. Most metastases occur within the first three years of a melanoma diagnosis, the most common locations for metastases are: lymph nodes, lung, liver, bone and brain. Patients diagnosed with melanoma must be followed for life not only because of risk of recurrence of their melanoma, but also because they are at increased risk of another primary melanoma. 32
  • 34. Question 1 of 3 What is the most likely diagnosis? A. Large congenital melanocytic nevus B. Becker’s nevus C. Melanoma D. Acanthosis nigricans L ARGE BROWN HAIRY PATCH A 19 year-old male presents with a several year history of an increasingly evident hyperpigmented and hypertrichotic area to his left upper chest. The lesion is asymptomatic. He is healthy and takes no medications. E. Tinea versicolour Check Answer Diagnosis & Information 33
  • 35. in the majority of cases a Becker’s nevus is considered a benign process. However, there have been a small number reported cases of melanoma within a Becker’s nevus. The diagnosis is usually clinical, although a skin biopsy can provide a histopathological diagnosis and helps distinguish it from other clinical entities, all of which are benign. Patients should be informed that this is a benign entity that does not require treatment except for cosmetic reasons. Patient concerns relate to the hyperpigmentation and/or the hypertrichosis. At present, it is quite difficult to remove the hyperpigmentation, although hypertrichosis can be safely and effectively removed by using laser hair removal or electrolysis. Clinical Pearl Diagnosis: Becker’s nevus A Becker’s nevus is an organoid nevus with hamartomatous elements, not a melanocytic nevus, predominantly affecting males. The nevus typically first appears as an irregular pigmentation (hyperpigmentation) on the torso or upper arm (though other areas of the body can be affected), and gradually enlarges irregularly, becoming thickened and often with hypertrichosis. There is an association with ipsilateral hypoplasia of breast and skeletal anomalies including scoliosis, spina bifida occulta, or ipsilateral hypoplasia of a limb that has been termed the Becker nevus syndrome, a sporadic condition. Other than the syndrome, • This benign late appearing birthmark is most common on the upper trunk and patients should be reassured as to its benign nature, and that currently we can clear the hypertrichosis, but that we are likely a few years away from being able to remove the hyperpigmentation with laser. “The skin is a diagnostician’s delight. Everything to be named is in full view. One has but to look, and recognize.” 
 ~ Drs. Walter & Dorinda Shelley, American Dermatologists, Advanced Dermatologic Diagnosis 34
  • 36. Question 1 of 3 What is your diagnosis? A. Alopecia secondary to irondeficiency B. Androgenetic alopecia C. Alopecia areata R OUND , BALD PATCHES ON SCALP A 29 year-old male is concerned with three round areas of hair loss to his posterior scalp that he has had for the previous two months. The areas are asymptomatic and he is otherwise healthy, though he hasn’t had a blood test in years. He had a similar round lesion on his scalp years ago that resolved on its own over a few months. D. Telogen effluvium E. Tinea capitis Check Answer Diagnosis & Information 35
  • 37. round, well-defined, bald patches with “exclamation-mark hairs” at the periphery. AA has an unpredictable onset but most commonly affects the scalp. However, any of the hair-bearing areas of the body can be affected, including the eyebrows, eyelashes, beard, or even the body. Interestingly, there may also be nail pitting (similar to that found in psoriasis). It is typically asymptomatic, though occasionally mildly pruritic preceding the lesion development. Entire scalp involvement is termed alopecia totalis, while hair loss affecting the entire body is termed alopecia universalis. Diagnosis: Alopecia areata Alopecia areata (AA) is a non-scarring autoimmune condition that most commonly presents in young adulthood. AA, which affects men and women equally, affects 1.5-2% of the general population at some point in their lives. Most people with AA are healthy individuals, though they do have a higher risk of atopy, and autoimmune conditions such as thyroid disease and vitiligo; family members may have autoimmune conditions such as insulin-dependent diabetes, rheumatoid arthritis, thyroid disease or systemic lupus erythematosus. Although not life threatening, it can have a profound psychosocial impact on individuals. Lesions clinically present as one or more The differential diagnosis of AA should include other non-scarring alopecias such as telogen effluvium, androgenetic alopecia, and even tinea capitis. Occasionally a scalp biopsy or referral to a dermatologist is required to ascertain the diagnosis. Other autoimmune conditions including atopic dermatitis, vitiligo, thyroid disease, and pernicious anemia have been associated with AA, as has iron deficiency, and thus are often screened for on blood tests. The treatment of choice is intralesional triamcinolone acetonide (Kenalog®, 2.5-5mg/ml) injected in multiple sites 1cm apart on the affected areas every 4-6 weeks until regrowth. Hair growth usually becomes apparent in four weeks. Local skin atrophy, the predominant side effect, can be minimized by taking care to inject into the mid-dermis, rather than into the more superficial epidermis or the lower subcutaneous fat. Potent topical steroids are also used, as well as topical minoxidil (Rogaine®) and contact sensitizers such as diphencyclopropenone (DPCP) for more widespread involvement. Hair prostheses should be recommended for widespread involvement. 36
  • 38. Clinical Pearl • In children with alopecia areata, in light of the often selflimited nature of this condition and the pain factor, starting treatment with a potent topical steroid often makes more sense than going straight to intralesional cortisone. “Hair brings one’s self-image into focus; it is vanity’s proving ground. Hair is terribly personal, a tangle of mysterious prejudices.”
 ~ Shana Alexander, former Journalist and Commentator on 60 Minutes 37
  • 39. Question 1 of 3 What is your diagnosis? A. Impetigo B. Angular cheilitis C. Hand, foot and mouth disease D. Herpangina S ORES AT CORNER OF MOUTH A 24 year-old female presents with painful lesions at the corner of her mouth. Every few months, she develops tingling, burning and erosions to the area and is quite frustrated with topical antiviral therapy. E. Herpes simplex Check Answer Diagnosis & Information 38
  • 40. such as those lining the eyes, vagina, cervix, or inside of the mouth. The skin around the blisters is often red. The diagnosis of HSV infection may be made clinically, but laboratory confirmation is recommended in patients presenting with primary or suspected recurrent infection. The gold standard of diagnosis is viral isolation by tissue culture, although this process can take as long as four to five days, and the sensitivity rate is only 70% to 80%. Despite these limitations, viral culture is still the diagnostic test of choice for HSV skin infections. Serologic testing is extremely sensitive but is not helpful during primary infection because of the delay in antibody development. Polymerase chain reaction enzyme-linked immunosorbent assay (PCR-ELISA) is extremely sensitive (96%) and specific (99%) but expensive. For this reason, it is not used for the diagnosis of skin lesions but is the laboratory test of choice for diagnosing HSV encephalitis. Diagnosis: Herpes simplex Herpes simplex virus (HSV) is a DNA virus presenting as grouped vesicles on an erythematous base. Typically infections will be recurrent in the same location. While oral herpes labialis is usually caused by HSV-1, genital involvement is more commonly due to HSV-2, although we now see both strains in both locations. Transmission occurs when an actively shedding individual (with clinical lesions or not) contacts mucous membranes which may be open or abraded. The virus then travels to sensory dorsal root ganglion where latency is established. Various stimuli can elicit outbreaks, including: trauma, fever, stress, hormonal fluctuations, and immunosuppression (e.g. from UV radiation). An eruption of tiny vesicles appears on the skin or on the mucous membranes, Susceptible patients without existing antibodies to HSV develop primary infection after first exposure to HSV. Any recurrence of HSV is termed “recur-rent infection.” About 20% to 40% of the general population experience recur-rent infection of the lips or perioral region. Herpes viruses establish lifelong infections and the virus cannot currently be eradicated from the body. Treatment usually involves antiviral drugs (acyclovir, valacyclovir, famciclovir; valacyclovir (Valtrex®) has the highest percentage bioavailability at 70%) that interfere with viral replication, reducing the physical severity of outbreak-associated lesions and lowering the chance of transmission to others. Resistance to antiviral drugs in immunocompetent individuals is rare. Treatment is most effective 39
  • 41. if started early (within 12 hours of the start of symptoms such as burning or tingling sensation is ideal, although within 72 hours is part of the formal indication) and can help symptoms resolve a day or two sooner (“episodic therapy”). Studies of vulnerable patient populations have indicated that daily use of antivirals can reduce reactivation rates. For people who have frequent, painful attacks, reactivation rates can be reduced by continuous therapy (“chronic suppressive” or “maintenance therapy”) with antiviral drugs. Docosanol (Abreva®) is an over-the-counter cream used for recurrent oral-facial herpes simplex episodes (cold sores or fever blisters). It is used 5 times per day for up to 10 days. Acyclovir 5% cream (Zovirax®) is a prescription antiviral topical agent used to treat cold sores. It is used 5 times per day for 4 days once tingling sensation or erythema is noticed to the mouth region. Oral antiviral therapy is considered significantly more effective than topical therapy. Analgesic medications are also commonly used to manage painful lesions. Clinical Pearl • Episodic therapy is sufficient for patients with only the occasional herpes simplex eruption, though for patients with frequent eruptions or who want to lessen the risk of transmission to others, chronic suppressive therapy is advised. “An inefficient virus kills its host. A clever virus stays with it.” ~ Professor James Lovelock, Independent scientist and ecologist 40
  • 42. Question 1 of 3 What is the most likely diagnosis? A. Verruca vulgaris B. Squamous cell carcinoma C. Basal cell carcinoma D. Prurigo nodularis R APIDLY EVOLVING HAND PLAQUE A 78 year-old female presents with a three month history of a rapidly growing large plaque with central crust on her dorsal hand, and which is occasionally tender. She has mild hypertension, but is otherwise healthy. She has no history of skin cancer, but has had “things burned off.” E. Keratoacanthoma Check Answer Diagnosis & Information 41
  • 43. KAs typically affect elderly Caucasians, especially men. Several variants of this tumor exist, including: eruptive keratoacanthoma of Grzybowski (multiple non-involuting KAs), and multiple Ferguson-Smith keratoacanthoma (rare, autosomal dominant, self-healing, affecting young adults). Treatment options are largely surgical. If left untreated, many KA lesions can resolve spontaneously over a period of a few months. However, due to a small metastatic potential as evidenced by rare instances of metastases or invasion (possibly misdiagnosed SCC), surgical excision of the lesion is warranted in many cases. Occasionally systemic retinoids are employed for multiple KAs. For poor surgical candidates or those with lesions that due to size and location are difficult to excise, intralesional methotrexate, 5FU or bleomycin can be effective. Aggressive liquid nitrogen cryotherapy can also be considered and may need to be repeated. Clinical Pearls Diagnosis: Keratoacanthoma (KA) A keratoacanthoma (KA) is a common, low-grade tumor that can clinically resemble a squamous cell carcinoma. Some experts believe KA to be a variant of squamous cell carcinoma, and thus suggest similar treatment. A keratoacanthoma is characterized by its rapid growth velocity (over weeks to months). There is often a solitary, firm skin to red-coloured papule with a central crateriform keratin plug (“volcano” appearance). Etiologic factors for this tumor are multifold, including: ultraviolet radiation, immunosuppression, smoking, human papilloma virus, tar or pitch exposure, trauma, and genetic factors. • Although some KAs can self-resolve even if left alone, because the lesion is often unsightly or uncomfortable, and because of the potential that the lesion is actually a squamous cell carcinoma, KAs are typically treated for cure. Additionally, the scar left after spontaneous resolution is typically not as cosmetically acceptable as a surgical scar. • A rapid growing scaly tumour that looks like a mini-volcano is likely a KA. 
 “People don’t get checked enough. What scares me is that a lot of people with skin cancer came in for something different and I just happened to see the tumor. ” 
 ~ Dr. Quang Le, American Dermatologist 42
  • 44. Question 1 of 3 What is the most likely diagnosis? A. Lichen planus B. Lichen simplex chronicus C. Pityriasis rosea D. Pustular psoriasis I TCHY PAPULES ON WRISTS A 33 year-old male presents with numerous pruritic papules on his wrists, shins and ankles. The patient finds the rash very bothersome and recently has difficulty sleeping. He takes occasional migraine medications, and has no drug allergies. E. Nummular eczema Check Answer Diagnosis & Information 43
  • 45. Lesions slowly develop over weeks and months, often starting on the wrists. Pruritus is common and worse in the hypertrophic form (often on the shins). Oral lesions can be asymptomatic or cause burning and pain. Most cases will resolve within two years, although oral involvement often implies a longer duration. Along with cutaneous involvement, scalp (lichen planopilaris can cause scarring and permanent alopecia), nails (10% of patients; nail plate thinning, grooving and ridging, scarring), oral (tongue, buccal) and genitals (vulva, glans) can be affected. Diagnosis: Lichen planus Lichen planus (LP) is a pruritic papulosquamous disease of planar, polygonal, and purple/violaceous papules (5 P’s). Common sites of involvement include the flexor surfaces of the arms and wrists, shins, ankles, genitalia, and oral mucosa (lacy white “Wickham striae”). There is no gender or racial predilection, and while LP can occur at any age, those age 30-60 years-old are most commonly afflicted. Several clinical subtypes have been observed: hypertrophic, atrophic, erosive, follicular, annular, linear, vesiculobullous, lichen planus pemphigoides, and actinic. The diagnosis of LP is often clinical, although histopathology may be required to confirm the diagnosis. LP appears to be an idiopathic immunologically-mediated inflammatory disorder. Stress may play a role in triggering this condition. Different variants exist, which have different presentations and symptomatology. Hypertrophic LP is particularly pruritic, and often affects the shins. The oral erosive form of lichen sclerosus has been associated in some cases with Hepatitis C viral infection. Ulcerative lesions of the mouth and vulva, unlike the cutaneous lesions, have a small risk of malignant transformation into squamous cell carcinoma. Other conditions with altered immunity may co-exist with LP, including alopecia areata, vitiligo, lichen sclerosis and myasthenia gravis. LP is a self-limited condition that usually resolves within 1-2 years. Most patients are treated symptomatically and with potent topical steroids. Oro-genital involvement can benefit from topical calcineurin inhibitors, such as tacrolimus (Protopic®). Less commonly, oral metronidazole (Flagyl®), oral acitretin (Soriatane®), phototherapy, and systemic steroids are employed by dermatologists for resistant or generalized LP. 44
  • 46. Clinical Pearl • Both psoriasis and lichen planus commonly present in men on the glans penis (remember the P’s of lichen planus: purple, polygonal, planar, pruritic papules….and penis!). While STDs are important to rule out in lesions affecting the glans, remember these two immunologic conditions as well. “Ordinarily, lichen planus is as easily recognizable as an old friend. He greets you with an itch and wears iridescent purple patches in all the old familiar places on wrist and ankle….then with experience you will learn to recognize your friend by his masks. You will come to know him sans itch, sans papule, and sans Wickham’s striae.” 
 ~ Drs. Walter & Dorinda Shelley, American Dermatologists and authors of Advanced Dermatologic Diagnosis 45
  • 47. Question 1 of 3 The most likely diagnosis is? A. Tinea corporis B. Cutaneous lupus erythematosus C. Chronic form of nummular dermatitis S CALY , ERYTHEMATOUS RASH ON ELBOWS A 63 year-old woman presented with a scaly, erythematous rash on her elbows and buttocks. She has been using daily moisturizers and 1% hydrocortisone cream without much clinical improvement. Her past medical history includes hypertension and dyslipidemia. She is currently taking ramipril (Altace®), atorvastatin (Lipitor®), and a multivitamin on a daily basis. D. Psoriasis vulgaris 
 (mild-to-moderate type) E. Mycosis fungoides Check Answer Diagnosis & Information 46
  • 48. About 35% of patients with psoriasis have a family history of the disease. Several environmental factors can trigger psoriasis in susceptible individuals: infection (most commonly streptococcal infection); trauma to the skin (surgery, sunburn, scratching; known as “Koebner phenomenon”); drug reaction (e.g., lithium, beta blockers, anti-malarial drugs, non-steroidal antiinflammatory drugs, and glucocorticoids); and stress. Diagnosis: Psoriasis vulgaris 
 (mild-to-moderate type) Psoriasis is a chronic, recurring inflammatory disease that can affect the skin, scalp, nails and joints. The typical lesions are pruritic, erythematous, and present as well-demarcated papules and plaques with silvery-white scales. Psoriasis affects about 2% of the population and ranges in severity from mild to severe; patients with moderate to severe disease can experience a significant impact on quality of life. It affects men and women equally. The age of onset of psoriasis follows a bimodal distribution (peaks between ages 20 to 30 years and again between the ages of 50 to 60). Both genetic and environmental factors have been implicated in the pathophysiology of psoriasis. The clinical presentation of psoriasis varies depending on the morphologic subclass. Plaque psoriasis (Psoriasis vulgaris) is the most common subtype and is usually concentrated on the extensor surfaces (i.e., elbows, knees, and lumbar back), scalp, genital areas, palms, and soles. Removal of scale causes sites of punctate bleeding (Auspitz sign), a diagnostic sign of historic note. Therapy varies depending on disease severity, as determined by the degree of body surface area involvement (BSA). Mild psoriasis involves <2% of the BSA, moderate psoriasis covers 2% to 10% of the BSA, and severe psoriasis covers >10% of the BSA. However, the vast majority of patients (approximately 80-90%) presents with relatively mild disease and have only limited involvement of the skin, which can be well-controlled with topical therapy alone. Patients with scalp, face, genital, and palmoplantar involvement in particular can have a very significant impact on their quality of life. Diagnosis of psoriasis is mainly made on clinical grounds. History and physical examination are usually diagnostic and should include a screen for arthritis because up to 30% of patients with psoriasis have or will develop joint involvement (psoriatic arthritis). Skin biopsy can be helpful in difficult cases. The differential diagnosis includes pityriasis rosea, seborrheic dermatitis, lichen simplex chronicus, tinea infections, 47
  • 49. psoriasiform drug eruptions, mycosis fungoides, and nummular eczema. Mild-to-moderate psoriasis is typically treated using topical agents. Useful topical preparations include those including corticosteroids, vitamin D analogues, combinations of these, and calcineurin inhibitors. Corticosteroids remain the mainstay of topical treatment for mild-to-moderate psoriasis. In choosing an appropriate corticosteroid potency and vehicle, the physical location being treated and patient preference should be considered. Psoriatic patients with thick, chronic plaques often require treatment with the highest potency corticosteroids (+/LCD tar or salicylic acid) and/or intralesional triamcinolone acetonide. Topical vitamin D modulators are among the most widely used medications for the treatment of mild-to-moderate psoriasis. Dovonex® (calcipotriol ointment and cream) and Silkis® (calcitriol 3 µg/g) ointment are synthetic topical vitamin D analogs with a great safety profile. Dovobet® (calcipotriol/ betamethasone proprionate) ointment combines the benefits seen in both a topical vitamin D analog and a topical corticosteroid. Mechanistically, this refers to the anti-proliferative/ prodifferentiating effects of the vitamin D analog component and the anti-inflammatory effects of the corticosteroid component. Clobex® spray (clobetasol propionate) is another good and elegant option for plaque psoriasis. of calcineurin inhibitors in some patients are a burning sensation and pruritus with initial treatments; however, the discomfort generally diminishes with ongoing use. The aforementioned topical therapies can be applied once or twice daily for up to 8 weeks. If insufficient improvement is noted, consider referral to a dermatologist. For recalcitrant mild-to-moderate or for more extensive psoriasis, psoralen plus ultraviolet-A topical therapy administered 2 to 3 times a week or more commonly narrow-band (311 nm) ultraviolet-B therapy are effective options and typically covered under provincial plans. Several systemic agents have been used as therapy for moderate psoriasis not controlled by topical therapy alone. Methotrexate, acitetrin, and less commonly cyclosporine are most widely used. Rotating ultraviolet light and systemic medications can help prevent toxicity from continuous use of any one therapy alone. Increasingly, biologic therapies (e.g. adalimumab, etanercept, infliximab, and ustekinumab) are also being used for moderate plaque psoriasis. There are two topical preparations of calcineurin inhibitors: Protopic® (tacrolimus ointment 0.03% and 0.1%) and Elidel® (pimecrolimus cream 1.0%). Tacrolimus and pimecrolimus have been used in areas of skin where greater topical penetration is improved, such as in flexural or facial skin. The main side-effects 48
  • 50. Clinical Pearls • Make sure to use the right vehicle in the right location. For instance, for the scalp or other hair bearing areas, lotions, foams or gels are great vehicles, while for very thick or fissured plaques of psoriasis, an ointment is a better fit. Also, ask your patients if they have a specific preference. • Psoriasis skin is very thick, so patients need to understand that although “steroids thin the skin,” in psoriasis this is a very wanted side effect! • For patients asking about more natural or lifestyle options, there is evidence to support the benefits of weight reduction, smoking cessation, reducing alcohol, and consuming more fish or fish oil capsules. • For patients with a significant impact on their quality of life, consider more aggressive therapeutic options and/or referral to a dermatologist. “Psoriasis is an acronym for a Pitting, Scaling, Ongoing, Red, Inflammatory, Anguishing, Symmetrical, Inherited Skin disease.” 
 ~ Drs. Walter & Dorinda Shelley, American Dermatologists and authors of Advanced Dermatologic Diagnosis 49
  • 51. Question 1 of 3 What is the most likely diagnosis? A. Fordyce glands B. Flat warts C. Xanthelasma D. Syringomas P ERIOCULAR PAPULES A 34 year-old female is bothered by the appearance of papules under her eyes that she has had for several years. She mentions that an aunt had similar asymptomatic, flesh-coloured papules which increased in number over time. She is healthy and takes no medications. E. Milia Check Answer Diagnosis & Information 50
  • 52. include the axilla, chest, abdomen, and genitals. Syringomas have been associated with Down syndrome. Where clinical uncertainty as to a diagnosis exists, a skin biopsy or referral to a dermatologist can be beneficial. Patients should be reassured as to the benign nature of these lesions. Treatment is primarily for cosmetic reasons only, with the goal of having complete destruction of lesions without evidence of scars. Syringomas can be treated with local excision, electrodessication, dermabrasion or cutaneous resurfacing with a carbon dioxide or erbium-yag laser. Clinical Pearl Diagnosis: Syringomas • Syringomas are benign, cosmetically unappealing skincoloured to yellowish small papules most commonly found in females, especially in my experience in Asian skin. The lower eyelids have many more lesions than the upper eyelids. Syringomas, considered benign adnexal tumors, are fairly common lesions more commonly affecting females, and predominantly of cosmetic concern. Lesions typically first appear around the time of puberty. • Even with treatment, patients should be warned that lesions can recur and/or new lesions may develop over time, so multiple treatments would be typical over one’s lifetime. Syringomas are asymptomatic flesh-colored to yellow small dermal papules that can also appear translucent. The lesional surface is either flat-topped or round, and the lesions typically are quite small, having a diameter of less than 4mm. Syringomas are distributed symmetrically in clusters, usually involving the lower eyelids and upper cheeks. Other less commonly affected areas “You’ll never get the big picture without looking at the little details.” ~ Drs. Walter & Dorinda Shelley, American Dermatologists and authors of Advanced Dermatologic Diagnosis 51
  • 53. Question 1 of 3 What is the most likely diagnosis? A. Verruca vulgaris B. Papular dermatitis C. Molluscum contagiosum D. Corns F INGER PAPULES E. Acquired digital fibrokeratomas A 32 year-old male presents with a seven month history of papules growing and multiplying on his hand. They are asymptomatic, but unsightly. Check Answer Diagnosis & Information 52
  • 54. Warts are transmitted by direct or indirect contact, and disruption of the epithelial barrier is a predisposing factor. A subset of HPV types has been associated with the development of malignancies: HPV 6, 11, 16, 18, 31, and 35; malignant transformation is most common in those with genital warts or those who are immunocompromised. There are now two HPV vaccines authorized for use in Canada: Gardasil® and Cervarix®. Gardasil® is approved for use in females and males aged 9-26, and provides protection against four HPV types: HPV-6, HPV-11, HPV-16 and HPV-18. Cervarix® was recently approved for use in Canada for females aged 10 to 25. At this time Cervarix® has not been approved for use in males in Canada. Cervarix® provides protection against the two HPV types: HPV-16 and HPV-18. Diagnosis: Verruca vulgaris (Warts) Cutaneous warts, one of the most common skin diseases, are a frequent presenting complaint to family practice and dermatology clinics. Warts are benign proliferations of skin and mucosa confined to the epithelium caused by one of the more than 150 types of the double-stranded DNA human papilloma virus (HPV). Common, plantar and flat warts are cutaneous manifestations of the human papillomavirus (HPV). Certain HPV types tend to occur at particular anatomic sites; however, warts of any HPV type may occur at any site. Clinically, HPV manifests itself as either: common warts, flat warts, genital warts, or palmoplantar warts. The treatment of warts poses a therapeutic challenge for physicians as no general consensus on monotherapy to achieve complete remission in all patients has been reached. As a result, many different approaches to wart therapy exist with varying mechanisms of action and efficacy. A ‘watch-and-wait approach’ is the first option to consider, since 65% of warts are believed to regress spontaneously within two years. Non-treatment in patients with extensive, spreading, or symptomatic warts is not advised. The first line of therapy is the topical agents, particularly salicylic acid, which is effective for non-genital warts. Many salicylic acid preparations are available over the counter (e.g. Soluver plus®, Duofilm™) at concentrations up to 40%, can be applied at home, and have cure rates from 70-80%. Dermatologists may prescribe even stronger compounded acid mixtures containing trichloroacetic acid, salicylic acid and lactic acid. Topical vitamin A acid/retinoids can also be considered in the therapeutic armamentarium. 53
  • 55. There are several topical agents that are applied only in the physician’s office, and they include catharidin and powerful sensitizing agents such as diphencyprone in a treatment referred to as immunotherapy. Cryotherapy destroys warts by thermal cytolysis. It can be used to treat warts in any location. An experienced physician applies liquid nitrogen with a cotton applicator or the more aggressive cryospray. Cryotherapy has been shown to be superior in the treatment of common warts. 5fluorouracil (5-FU), such as Efudex®, is a chemotherapeutic agent that interferes with DNA and RNA synthesis, and both topical and intralesional compounds have been used to treat genital warts. Imiquimod (Aldara®, Zyclara®) has also been used for hand warts with some success, although it is more useful for mucosal warts where absorption is much greater. For persistent and refractory warts, intralesional injections, most commonly Bleomycin or candida antigen, can be performed by an experienced physician. Clinical Pearl • Make clear to your patients that there is no quick and easy fix as far as warts are concerned. • Evidence-based medicine for non-mucosal warts best supports liquid nitrogen cryotherapy and salicylic acid preparations. • Consider oral zinc therapy as an immune booster for stubborn warts – start at 50mg/day with food (studies show it can be increased as tolerated up to 600mg/day). “Of all the futile disorders of the skin, it would be hard to find any that are regarded with greater contempt by the lay public and yet capable of resisting a greater variety of treatment than the group of papillary lesions commonly known as warts.” 
 ~ Dr. W.W. Lempiere, Australian Dermatologist The treatment of warts continues to pose a therapeutic challenge. Many different approaches to wart therapy exist with varying mechanisms of action and efficacy. The treatment options are extremely heterogeneous and no single treatment or group of treatments is standard. Patients are often dissatisfied with recurrence, as a therapeutic regimen to achieve complete remission in all patients has not yet been developed. Additional clinical trials of the many therapeutic options will help to clarify the best treatment choices for cutaneous warts. 54
  • 56. Question 1 of 3 What is the most likely diagnosis? A. Subungual wart B. Psoriatic nail C. Onychomycosis D. Traumatic nail changes T HICK , YELLOW TOENAILS A 56 year-old male has slowly changing nails over the past 10 years. They have gotten thicker, yellow in colour, and occasionally cause him discomfort. He finds cutting his toenails to be increasingly difficult. E. Pseudomonas nail infection Check Answer Diagnosis & Information 55
  • 57. dermatophyte molds. However, the clinical appearance is indistinguishable between the different fungal species. The main subtypes include: distal lateral subungual onychomycosis (DLSO), white superficial onychomycosis (WSO), proximal subungual onychomycosis (PSO), and candidal onychomycosis. Clinically, onychomycosis manifests with an increased nail thickness, an opacified nail plate, subungual thickening, and/or onycholysis. There is also often discoloration of the nail ranging from white to yellow to brown in colour. Diagnosis is usually clinical, although the nail changes of psoriasis or lichen planus and the effects of aging can have a similar appearance. Further testing such as direct microscopy with 20% potassium hydroxide (KOH) testing can be beneficial, and a fungal culture is often used to identify the causative organism. Diagnosis: Onychomycosis Onychomycosis is a fungal infection that most commonly affects the toenails. It can affect any component of the nail unit, including the nail matrix, nail bed, or nail plate. The majority of the time, the concern is primarily cosmetic in nature, but occasionally, there can also be discomfort and pain. The prevalence of onychomycosis is approximately 6% and it accounts for half of all nail disorders. Onychomycosis is uncommon in childhood, but is seen with prevalence rates up to 90% in the elderly. The most common microorganism implicated is the dermatophyte Trichophyton rubrum. Onychomycosis is caused by 3 main types of fungi: dermatophytes, yeasts, and non- Topical therapy is beneficial in mild cases involving the very distal nail plate and when few nails are involved. Topical therapy is also considered in combination with systemic therapy or where the patient is on multiple oral medications. The best topical therapy option is likely ciclopirox olamine 8% nail lacquer solution (Penlac®). Ciclopirox lacquer is usually applied to the affected nails once daily at bedtime or at least 8 hours before washing the affected area. Apply daily over the previous coat. Once a week, remove the lacquer with rubbing alcohol. Loose nail material should be filed and nails trimmed, as required. Treatment of onychomycosis (especially multiple nails or of long duration) is largely systemic with terbinafine (Lamisil®) considered by most to be the drug of choice. Less commonly, itraconazole pulse therapy or nail avulsion is employed. Not treating the nails is a common and important option to discuss. 56
  • 58. Clinical Pearls • Onychomycosis and tinea pedis are often important to treat in diabetic patients since the fissures or maceration caused by tinea can provide an entry point for bacteria, and ultimately risk for cellulitis, sepsis, or amputation. • For patients with recurrent tinea cruris (“jock itch”), always check their toenails as there is often a concomitant onychomycosis. “You can tell a lot from a person’s nails. When a life starts to unravel, they’re among the first to go.” 
 ~ Ian McEwan, critically-acclaimed author and Man Booker Prize winner 57
  • 59. Question 1 of 3 What is the most likely diagnosis? A. Scalp psoriasis B. Severe seborrheic dermatitis C. Mycosis fungoides S CALY , ERYTHEMATOUS PLAQUES ON HEAD A 54 year-old man comes to your clinic with a longstanding history of ‘dandruff’ that is pruritic and he is unable to get under control. He has tried using many shampoos advertised on television and recommended by his pharmacist but has seen no clinical improvement. He also has scaly plaques to his elbows and knees. He is otherwise healthy and takes no medications. D. Tinea capitis E. None of the above Check Answer Diagnosis & Information 58
  • 60. white or gray scale and may extend beyond hair margins, affecting the forehead, neck, ear and retroauricular area. Quality of life can be severely compromised, particularly if lesions are visible, scales fall onto clothes, and when pruritus is intense. Hair loss secondary to poorly managed scalp psoriasis can occur. Thick accumulated scale and hair, particularly with very thick hair, may be obstacles to medications reaching the scalp. The diagnosis of scalp psoriasis is most often made clinically. In some cases, it may be difficult to differentiate between scalp psoriasis and seborrheic dermatitis, but the presence of psoriasis in other locations may aid its diagnosis in the scalp. Localization of disease past the scalp margins and sharply demarcated plaques may help differentiate scalp psoriasis from seborrheic dermatitis, as seborrhea usually does not extend beyond the scalp margins but may be found in the central facial region or central chest. Diagnosis: Scalp psoriasis Psoriasis affects approximately 2% of the population, and in 50-80 percent of cases, the scalp is affected. Scalp involvement may occur in isolation, with plaque psoriasis located elsewhere (most common), or with erythrodermic, pustular or guttate psoriasis, and/or psoriatic arthritis. In 25-50 percent of affected patients, psoriasis presents initially on the scalp. Family history may predispose patients to scalp psoriasis. Mild forms of scalp psoriasis involve minimal scaling, while severe disease may be associated with thick, crusted plaques that are frequently associated with moderate to severe itch. Classic scalp lesions are asymmetric, sharply demarcated, covered with silver- Scalp psoriasis is a chronic inflammatory disease and cure is not a realistic treatment expectation. Instead, the clinician’s focus should be on enhancing the patients’ skin-related quality of life. Non-pharmacological physical therapies which include moist occlusion to plaques and gentle debridement of scalp scaling are useful in some patients. Occlusion therapy involves first applying skin products, such as moisturizers to the skin then wrapping the skin with plastic wrap and tape. Occlusion helps keep the area moist and increases the effectiveness of medicated creams. Topical agents are the mainstay of pharmacologic treatments, with clinically proven efficacy. Scalp psoriasis can be managed with topical preparations of tar and salicylic acid, such as Sebcur/ T™ (10% tar with 4% salicylic acid) and Neutrogena® T/Gel® (0.5% tar) medicated shampoos, Derma-smoothe/FS® scalp oil (fluocinolone topical oil) overnight with shower cap, Dermovate® 59
  • 61. lotion (clobetasol 17 - propionate), or Diprosalic® lotion (0.05% betamethasone and 2% salicylic acid). The retinoic acid derivative Tazorac® (0.1% tazarotene gel), applied nightly, can be helpful for scalp psoriasis, but must be used in combination with steroids to prevent irritation. Newer topical agents for scalp use are being used currently with better patient satisfaction and compliance. Dovobet® gel is a new lipophilic gel specially formulated for the scalp and contains the active ingredients calcipotriol 0.005% (synthetic vitamin D analogue) and betamethasone dipropionate 0.05% (potent corticosteroid).” Clobex® (Clobetasol proprionate) 0.05% shampoo is also efficacious and safe for the management of moderate scalp psoriasis; it is often prescribed twice weekly (alternating with other psoriasis shampoos) and effectively helps to prevent the relapse of scalp psoriasis; this 15 minute shortcontact shampoo formulation can be safely utilized for extended periods and weaned as necessary. Topical therapies may be used in combination with sunlight or ultraviolet light (phototherapy) for moderate psoriasis that affects less than 20% of the skin surface. Brief exposures to ultraviolet B light (UVB), and less commonly UVA, improves psoriasis and treatments are usually conducted three times per week. As an alternative to topical application, corticosteroids administered through intralesional injection (e.g. Kenalog® (triamcinolone acetonide), 5-10 mg/mL) to recalcitrant scalp lesions may provide some patients with a reduction of symptoms, often lasting several months. Systemics used for management of psoriasis include cyclosporine (least common), methotrexate and acitretin; and although these therapies may be effective, each involves inherent limitations. Biologics have become an important modality for the treatment of moderate-to-severe chronic plaque psoriasis and may be an effective approach to management of difficult plaque psoriasis of the scalp. Current biologics on the market for psoriasis include the tumour necrosis factor-alpha blockers Enbrel® (etanercept), Remicade® (infliximab), and Humira® (adalimumab). One of the newest biologics is Stelara® (ustekinumab), a human monoclonal antibody directed against interleukin (IL)-12 and IL-23. Many new biologics are in the pipeline. Clinical Pearls • For stubborn scalp psoriasis, consider multi-modal treatment, meaning shampoo and overnight gel or lotion use. For very stubborn plaques, consider intralesional Kenalog®. • Scalp psoriasis can have a tremendous impact on quality of life and can be difficult to control, and so occasionally systemics or biologics are employed even if this is the only area affected by psoriasis. Phototherapy can also be considered, especially in those persons with little or no hair. “I have long had the idea that God created psoriasis and warts to teach dermatologists the merits of humility.”
 ~ Vincent Joseph Derbes, Dermatologist and former Editor, International Journal of Dermatology 60
  • 62. Question 1 of 3 What is the most likely diagnosis? A. Basal cell carcinoma B. Actinic keratosis C. Keratoacanthoma D. Irritated seborrheic keratosis L ARGE SCALY SCALP PLAQUE A 67 year-old male presents with a scaly plaque on his scalp. It has slowly enlarged in the past year, and is otherwise asymptomatic. He has a history of actinic keratoses, and worked in construction. E. Squamous cell carcinoma Check Answer Diagnosis & Information 61
  • 63. involvement and invasion of dermis). The diagnosis of squamous cell carcinoma is based on clinical suspicion and confirmed by a histopathological diagnosis on biopsy. Treatment options include surgical excision, as well as curettage and electrosurgery (similar cure rates). For well-differentiated small SCCs (<2cm), a 4mm margin is recommended, whereas in larger tumors in high risk sites require at least a 6mm margin. Larger SCCs and those that are poorly differentiated have a higher incidence of local invasion and metastases. Mohs micrographic surgery is indicated for aggressive subtypes of SCC, recurrent tumors, or highly aggressive features (e.g. SCC in a burn scar). Other treatment options include radiation and cryosurgery in patients with impaired clotting disorders or tumors in difficult to excise areas. Diagnosis: Squamous cell carcinoma (SCC) Squamous cell carcinoma is currently the second most common skin cancer in Caucasians, and the most common type in black individuals. SCC can develop in the setting of actinic keratoses, leukoplakia, lichen sclerosus, radiation dermatitis, chronic arsenic exposure, scars, burns, chronic ulcers or sinuses (e.g. hidradenitis suppurativa). Prevention of SCC is similar to preventative treatment of actinic keratoses (AKs). Effective measures include avoidance of excessive sun exposure, especially during peak day-time hours, avoidance of artificial sources of ultraviolet (UV) light such as tanning beds or prolonged UV treatments, wearing protective clothing, applying sunscreen with at least 30 SPF over sunexposed areas a few times per day. Additionally, proper management of AKs and aggressively managing other underlying conditions (e.g. lichen sclerosus) is important. There is thought to be a spectrum of keratinocyte malignant transformation: actinic keratoses (atypical keratinocytes in up to one-half of the epidermis), Bowen disease or SCC-in-situ (full epidermal keratinocyte atypia), and invasive SCC (full epidermal 62
  • 64. Clinical Pearls • An SCC on a dark-skinned individual will often present on the lower leg as a benign-appearing hyperpigmented scaly plaque. • Both coffee and Cox-2 inhibitors may prevent non-melanoma skin cancer like SCC. “Skin cancer, if caught early, can be treated fairly straightforwardly, and it’s not that big a deal. But once skin cancer has spread, effectively nothing works. So it’s particularly important to catch skin cancer early.”
 ~ Dr. Darrell Rigel, American dermatologist 63
  • 65. Question 1 of 3 What is your diagnosis? A. Impetigo B. Molluscum contagiosum C. Juvenile xanthogranuloma D. Milia P APULES ON THE ABDOMEN A 5 year-old boy was brought into clinic by his mother because of a concern regarding papules that were developing on his abdomen, chest and upper arms. These papules were mildly pruritic, and had been present for 2 months and new papules were still appearing. There were no sick contacts, and the child felt well otherwise. E. Common Warts Check Answer Diagnosis & Information 64
  • 66. typically occur on the chest, arms, trunk, legs, and face. Mucous membrane involvement is rare, and the palmoplantar area is spared. Patients with a history of atopic dermatitis are more prone to MC and may develop a large number of lesions. Approximately 10% of all patients will develop eczema around the lesions. In adults, MC is most commonly a sexually transmitted disease and presents as a few scattered lesions often limited to the perineum, genitalia, inner thighs, lower abdomen, or buttocks. MC in healthy children and adults is usually a self-limited disease, but may persist for several months and even up to a few years. Widespread, persistent, and atypical MC may occur in patients who are significantly immunocompromised or have acquired immunodeficiency syndrome (AIDS) with low CD4 T-lymphocyte counts. Diagnosis: Molluscum contagiosum Molluscum contagiosum (MC) is a viral infection of the skin caused by a DNA poxvirus called the molluscum contagiosum virus (MCV), which is a benign and generally self-limited viral infection. Skin lesions typically consist of multiple dome-shaped, pink to skin-colored papules 2-6 mm in diameter of which some lesions show the classic feature of umbilication. MC is usually asymptomatic, although individual lesions can occasionally be tender or pruritic. MC is encountered most commonly in children who become infected through direct skin-to-skin contact or indirect skin contact with fomites, as well as by auto-inoculation. Lesions Diagnosis is usually clinical and based on the distinctive central umbilication of the dome-shaped papule. If diagnosis is uncertain, papules can be biopsied and sent for histopathologic identification. Adult patients should be questioned about sexual history and where appropriate, evaluated for other concomitant STDs. Always consider testing for HIV infection in patients with large or facial MC lesions. Patients and their families should be educated as to the benign and self-limited nature of this condition and that treatment is not a necessity. For the most part, the main concern is temporary adverse cosmetic results and embarrassment. Most lesions resolve with no permanent residual skin defect; however, occasional lesions may produce a slightly depressed scar, especially if excoriated. Although treatment is not required, it can help reduce autoinoculation or transmission to close contacts and improve clinical appearance. More than one treatment session is frequently required. 65
  • 67. In healthy children, a major goal is to limit physical discomfort. Often, no medical intervention or minor direct trauma is appropriate. Cryotherapy or curettage (can apply topical anesthetic cream pre-procedure) are very effective therapies for older children. For younger children, Cantharidin applied carefully to the lesions and washed off after 15 minutes to 2 hours can be effective. Similarly, tretinoin/vitamin-A acid applied topically only to the lesions (e.g. with a toothpick) can be used. Another treatment option to consider, with variable benefit, is topical imiquimod (e.g. Aldara™, Zyclara™), a topical immune response modifier and a potent inducer of anti-viral interferons. “What perhaps should receive more attention is the effect of the treatment on the virus.” ~ Luc Montagnier, French virologist and 2008 Nobel Prize Winner (Discovery of HIV) In adults who are more motivated to have their lesions treated, cryotherapy or curettage of individual lesions is effective and well tolerated. In immunocompromised individuals, MC can be extensive and difficult to treat. The prognosis of patients with MC is generally excellent because the disease is benign and self-limited. In healthy patients, 1-4 treatments are usually effective to see complete resolution of the lesions. Clinical Pearl • Be careful when treating self-limited conditions such as molluscum or warts in individuals with more pigmented skin as the treatments can result in hyper or hypo-pigmentation which often bother the patient even more than the original problem and can have medico-legal ramifications. 66
  • 68. Question 1 of 3 What is his diagnosis? A. Linear IgA disease B. Herpes simplex C. Herpes zoster D. Dermatitis herpetiformis P AINFUL , VESICULAR RASH A 30 year-old male presents with a painful vesicular rash to his right lateral flank region. He has been experiencing generalized malaise and myalgias to the affected area for the past 2 days. E. None of the above Check Answer Diagnosis & Information 67
  • 69. Diagnosis: Herpes zoster Herpes zoster (shingles) is an acute, painful dermatomal dermatitis that affects approximately 10% to 20% of adults, often in the presence of immunosuppression. Herpes zoster is a viral infection caused by the same virus (varicella) that causes chicken pox. During the course of varicella, the virus travels from the skin and mucosal surfaces to the sensory ganglia, where it lies dormant for a patient’s lifetime. Reactivation often follows immunosuppression, illness, emotional stress, trauma, and irradiation or surgical manipulation of the spine, producing a dermatomal dermatitis. Herpes zoster is primarily a disease of adults and typically begins with pain and paresthesia in a dermatomal or bandlike pattern followed by grouped vesicles within the dermatome several days later. Occasionally, fever and malaise occur. The thoracic area accounts for more than half of all reported cases. When zoster involves the tip and side of the nose (cranial nerve V) nasociliary nerve involvement can occur (30%-40%). Great care is needed if the blisters involve the eye because permanent eye damage can result, and so an ophthalmology consultation should be strongly considered. Most patients with zoster do well with only symptomatic treatment, but postherpetic neuralgia is common in the elderly. Disseminated zoster is uncommon and occurs primarily in immunocompromised patients. Since 2008, a live, injectable, attenuated herpes zoster vaccine (Zostavax™) has been authorized for use in Canada for the prevention of herpes zoster (shingles) infection in adults aged 60 years and older. Since its inception, the vaccine has greatly reduced the number of cases of postherpetic neuralgia and reduced the severity and duration of pain and discomfort associated with shingles. Diagnosis is by clinical presentation, viral culture, or direct fluorescent antibody from a skin biopsy. Prompt treatment reduces the extent and duration of symptoms as well as the risk of chronic sequelae. Antiviral agents (such as acyclovir, valacyclovir and famciclovir) promote healing of cutaneous lesions and prevent or reduce the severity of postherpetic neuralgia. The recommended adult dose of valacyclovir (Valtrex®) to treat shingles is 1000 mg tid for 7 days. The treatment should be initiated within 72 hours of the onset of the rash for better management of clinical sequelae. Narcotic and non-narcotic topical and oral analgesics, anti-convulsants, and tricyclic 68
  • 70. antidepressants can be used for acute zoster-associated neuralgias. The use of oral corticosteroid therapy is controversial. Botulinum toxin (Botox®) is also effective for management of pain caused by postherpetic neuralgia. The analgesic effects are not dependant on changes in muscle tone. Clinical benefit with Botox® is seen for at least 14 weeks after it is given. Local anesthetics applied directly to the skin of the painful area affected by postherpetic neuralgia are also effective. Lidocaine (Xylocaine®) is available in a patch that has been approved for use specifically in postherpetic neuralgia. It can be used to the affected area every 4-12 hours as needed. Topical local anesthetics stay in the skin and therefore do not cause problems such as drowsiness or constipation. Capsaicin (Zostrix®) cream may be somewhat effective and is available over the counter. It can be applied to the affected area 3-5 times daily. However, patients often find that it causes a burning pain during application. Clinical Pearl • Be careful when treating self-limited conditions such as molluscum or warts in individuals with more pigmented skin as the treatments can result in hyper or hypo-pigmentation which often bother the patient even more than the original problem and can have medico-legal ramifications. “In making a dermatologic diagnosis, look for the distinctive face of the disease, not its featureless arms, legs and trunk.” ~ Drs. Walter & Dorinda Shelley, American Dermatologists and authors of Advanced Dermatologic Diagnosis 69
  • 71. Question 1 of 3 What is the diagnosis? A. Inverse psoriasis B. Atopic dermatitis C. Mycosis fungoides D. Contact dermatitis I TCHY , ROUGH PATCHES ON LEGS A 41 year-old woman presents with a 6 month history of pruritic lesions affecting the popliteal fossae. She notices the lesions are more painful and itchy in the winter. She has tried some over the counter moisturizers that have only slightly helped. She had asthma and mild exzema as a child but is otherwise healthy and takes no medications. She has an older sister with a history of eczema. E. Nummular eczema Check Answer Diagnosis & Information 70